You are on page 1of 35

VISIONIAS

www.visionias.in
ANSWERS & EXPLANATIONS
GENERAL STUDIES (P) TEST – 3209 (2021)

Q 1.C
• Bamiyan is situated in the high mountains of the Hindu Kush in the central highlands of Afghanistan.
The valley, which is set along the line of the Bamiyan River, was once integral to the early days of the Silk
Roads, providing passage for not just merchants, but also culture, religion and language.
• The region became a major cultural, religious and trade centre when Buddhist Kushan Empire spread. The
countries like India, China and Rome asked for passage through Bamiyan, Kushans developed a syncretic
culture. The monastic quality of the valley also reflects faith.
• The Bamiyan Buddhas were great examples of a confluence of Gupta, Sassanian and Hellenistic
artistic styles. They were dressed in the Roman draperies and were standing in two different mudras.
They are said to date back to the 5th century AD and were once the tallest standing Buddhas in the
world. Salsal and Shamama, as they were called by the locals, rose to heights of 55 and 38 metres
respectively, and were said to be male and female.
o Salsal means “light shines through the universe”; Shamama is “Queen Mother”.
o The statues were set in niches on either ends of a cliffside and hewn directly from the sandstone cliffs.
• In March 2001, the Taliban blew up the two monumental Buddha statues in Afghanistan’s Bamiyan Valley.
Following the fall of the Bamiyan Buddhas, UNESCO included the remains in its list of world heritage sites
in 2003, with subsequent efforts made to restore and reconstruct the Buddhas in their niches with the pieces
available.
• Now, two decades later, on the anniversary of the annihilation, the Bamiyan Buddhas have been brought
back to life in the form of 3D projections in an event called “A Night With Buddha”.
• Hence, option (c) is the correct answer.

Q 2.D
• Recent Context: The Ministry of Commerce recently launched an online platform to issue Certificate of
Origin to the exporters and other guidelines. The move is being taken to facilitate shipments during
lockdown.
• Certificate of Origin
o It is an instrument that establishes evidence on the origin of goods imported into any country. Hence,
statement 2 is not correct.
o These certificates are essential for exporters to prove where their goods come from and therefore stake
their claim to whatever benefits goods of Indian origin may be eligible for in the country of exports.
o The Certificate of Origin (CO) is not mandatory but may be required by some countries for all or only
certain products. In many cases, a statement of origin printed on company letterhead will suffice. The
exporter should verify whether a CO is required with the buyer and/or an experienced shipper/freight
forwarder. Hence, statement 1 is not correct.
o Some countries (i.e., numerous Middle Eastern countries) require that certificate of origin be notarized,
certified by local chamber of commerce and legalized by the commercial section of the consulate of the
destination country.
• There are two categories of Certificate of Origin:
o Preferential
§ It enable products to enjoy tariff reduction or exemption when they are exported to countries
extending these privileges.
§ Some of the Preferential arrangements/schemes under which India is receiving tariff preferences
for its exports are Generalised System of Preferences (GSP), SAARC Preferential Trading
Agreement (SAPTA), Asia-Pacific Trade Agreement (APTA), ASEAN-India Free Trade
Agreement.
1 www.visionias.in ©Vision IAS
§ Preferential arrangements/agreements prescribe Rules of Origin which have to be met for exports
to be eligible for tariff preference.
o Non-Preferential
§ These Certificates of Origins evidence origin of goods and do not bestow any right to preferential
tariffs. Hence, statement 3 is not correct.
• Ministry of Commerce has authorised various agencies to charge a fee for providing services relating to
issue of Certificate of Origin, including details regarding rules of origin, list of items covered by an
agreement, extent of tariff preference, verification and certification. n of eligibility.

Q 3.D
• Pattadakal, on the banks of river Malprabha, is about sixteen kilometres from Badami. This royal
commemorative Hindu site served as a setting for the coronation ceremonies of the Early Western
Chalukyan rulers.
• The Virupaksha temple, the largest of the Pattadakal group was constructed in 740 A.D. by the queen of
the Chalukyan King Vikramaditya II to commemorate the victory of her husband over Pallavas of
Kanchipuram. The temple situated within a rectangular walled enclosure, has a large court and a hall for
Nandi. The temple is dedicated to Siva.
• The sanctuary of the temple is surrounded by a circumambulatory passage. The interior of the temple is
richly carved especially the pillars, ceilings and lintels. The columns have sculptural reliefs showing
episodes from the Ramayana and the Mahabharata. The Surya figure on the ceiling shows a liveliness of
spirit, standing with Usha and Pratyusha in a seven-horse chariot driven by Aruna.
• Hence option (d) is the correct answer.

Q 4.C
• The Tropical Forest Alliance is a global public-private partnership dedicated to collaborative action to
realize sustainable rural development and better growth opportunities based on reduced deforestation and
sustainable land use management in tropical forest countries. The Alliance includes more than 150 partners
representing the private sector, governments, civil society organizations, indigenous peoples groups,
and multilateral organizations who are committed to reducing tropical deforestation associated with the
production of palm oil, soy, beef, and pulp and paper. Hence statement 3 is correct.
• The partners take voluntary action as per the mission of the alliance and are not legally binded. Hence
statement 2 is not correct.
• The funders of the alliance currently include the governments of the Netherlands, Norway, Germany and
the United Kingdom, as well as the Gordon and Betty Moore Foundation. The Tropical Forest Alliance was
founded in 2012 at Rio+20 after the Consumer Goods Forum (CGF), though hosted by World Economic
Forum, it is not a daughter body of the forum as such. The CGF partnered with the US government to
create the public-private alliance with the mission of mobilizing all actors to collaborate in reducing
commodity-driven tropical deforestation. Hence statement 1 is not correct.

Q 5.D
• At the beginning of the 18th century, Kerala was divided into a large number of feudal chiefs.
• The kingdom of Travancore rose into prominence after 1729 under king Martanda Varma, one of the
leading statesmen of the 18th century. He combined rare foresight and strong determination with courage
and daring. He subdued the feudatories, conquered Quilon and Elayadam, and defeated the Dutch and
thus ended their political power in Kerala. He established an independent state of Kerala with
Travancore as his capital. Hence statements 1 and 2 are correct.
• He organised a strong army on the western model with the help of European officers and armed it
with modern weapons. He also constructed a modern arsenal. Hence statement 3 is correct.
• Martanda Varma used his new army to expand northwards and the boundaries of Travancore soon extended
from Kanya Kumari to Cochin. He undertook many irrigation works, built roads and canals for
communication, and gave active encouragement to foreign trade.

Q 6.C
• First World Solar Technology Summit (WSTS) being organized by the International Solar Alliance (ISA)
on 8th September 2020 along with the Federation of Indian Chambers of Commerce and Industry (FICCI).
The summit was inaugurated by Prime Minister of India Narendra Modi. The main objective of the summit
is to pay attention to state-of-the-art technologies and next-generation technologies. Hence statement 1 is
correct.

2 www.visionias.in ©Vision IAS


• The theory of disruptive technologies is a brainchild of a Harvard Business Review (HBR) article written
by two Harvard faculty members in 2015. The disruptive (new and significantly different) technologies
aim to diversify the traditional application pattern of solar energy that is electricity generation.
• For eg. remote phones, switches on utility distribution systems, and over-water warning lights for airports.
The members of the international solar alliance has called for the research and adoption of Disruptive Solar
Technologies (DST) in WSTS. Hence statement 2 correct.

Q 7.A
• At a special session in Calcutta in September 1920, Congress approved a non-cooperation programme
till the Punjab and Khilafat wrongs were undone and Swaraj was established. The programme included
the boycott of government schools and colleges, law courts, foreign cloth and legislative council. The
programme advocated for the renunciation of government honours and titles too. Hence statement 1 is
correct.
• Congress lost faith in constitutional struggle, especially after the blatantly partisan Hunter Commission
Report, and thus thought that it was the correct time to launch the Non-Cooperation Movement on the basis
of Khilafat which would help in welding the Hindu Muslim unity too.
• Annie Besant opposed the idea of the Non-Cooperation Movement outrightly. There were certain
leaders apart from Annie Besant like Mohammad Ali Jinnah, BC Pal who left the Congress as they believed
in the constitutional and lawful struggle. Hence statement 2 is not correct.
• The spirit of defiance and unrest gave rise to many local struggles such as Awadh Kisan Movement, Eka
Movement, Mapilla Movement.

Q 8.A
• In order to enhance the skilling ecosystem in the country under the Skill India Mission, the Ministry of
skill development and entrepreneurship established National Council for Vocational Education and
Training (NCVET) through ‘notification’. Therefore, it is not a statutory body. NCVET has
subsumed the existing National Council for Vocational Training and the National Skill Development
Agency which were earlier. Hence statement 1 is correct and Hence statement 2 is not correct.
• It consists of minimum of six Members and a maximum of ten Members who are appointed by the central
government.
• Functions and powers of the council:
o recognize, monitor, discipline and de-recognize awarding bodies, assessment agencies; and skill
related information providers;
o recognize Skills Universities as a separate category of bodies for providing advanced vocational
education and training programmes and conducting research and development in skill education and
training,
o frame guidelines for the conditions of recognition and functioning of awarding bodies and
assessment agencies including their roles and responsibilities with respect to training bodies, consumer
of such services
o The Council may, in agreements to be entered with awarding bodies, create rights and obligations for
training bodies as a condition of recognition of awarding bodies.
o Therefore, it acts as an overarching skills regulator that regulates the functioning of entities
engaged in vocational education and training. Hence statement 3 is correct.
Q 9.D
• Recent Context: Goa government decided to limit the parade of Shigmo festivities, in three locations –
Panaji, Ponda and Mapusa.
• Shigmo Festival
o Shigmo is the celebration of a ‘rich, golden harvest of paddy’ by the tribal communities of Goa.
o Agricultural communities including the Kunbis, Gawdas, and Velips celebrate the festival that also
marks the onset of spring.
o Shigmo celebrations last over a fortnight in the months of Phalgun-Chaitra months of the Hindu
calendar that correspond with March-April every year.
o Dances: Folk dances like Ghode Modni (a dance of equestrian warriors), Gopha, and Phugadi.
o Shigmo is celebrated all over India but in different names:
§ North India - Holi.
§ Assam and Bengal - Dolyatra.
§ South India - Kamadahan.
§ Maharashtra - Shimga.
• Hence option (d) is the correct answer.
3 www.visionias.in ©Vision IAS
Q 10.A
• The Industrial Relations Code combines the features of three erstwhile laws — the Trade Unions Act, 1926,
the Industrial Employment (Standing Orders) Act, 1946, and the Industrial Disputes Act, 1947.
• It defines ‘workers’ to include, besides all persons employed in a skilled or unskilled, manual, technical,
operational and clerical capacity, supervisory staff drawing up to ₹18,000 a month as salary. Hence,
statement 3 is not correct.
• It introduces ‘fixed term employment’, giving employers the flexibility to hire workers based on
requirement through a written contract. Fixed term employees should be treated on a par with permanent
workers in terms of hours of work, wages, allowances and other benefits, including statutory benefits such
as gratuity.
• The Code says any establishment that employs 300 or more workers must prepare standing orders relating
to classification of workers, manner of intimating to them periods and hours of work, holidays, pay days
etc, shifts, attendance, conditions for leave, termination of employment, or suspension, besides the means
available for redress of grievances.
• It confers on the ‘appropriate Government’, that is the Centre or the State governments, the power to
exempt, with or without conditions, any industrial establishment or class of industrial establishments from
all or any of the provisions of the Code, if it is satisfied that adequate provisions exist to fulfil its
objectives. Hence, statement 2 is not correct.
• The Code prohibits strikes and lock-outs in all industrial establishments without notice. No unit shall
go on strike in breach of contract without giving notice 60 days before the strike, or within 14 days of giving
such a notice, or before the expiry of any date given in the notice for the strike. The Industrial Disputes Act,
1947, had placed such restrictions on announcing strikes only in respect of public utility services. However,
the present Code extends it to all establishments. Even the Standing Committee on Labour had favoured
limiting these provisions to public utilities. Hence, statement 1 is correct.

Q 11.D
• Olive ridley turtle- The coastal state of Odisha on the eastern coast of India near the mouth of the
Rushikulya river experiences one of the world’s largest mass nestings or arribada of the Olive Ridley turtle
during the months of October to April.
• Leatherback Sea turtles- are the largest living sea turtles, growing up to two metres and weighing as much
as 900 kg. They are known for their remarkable migrations, travelling several thousands of kilometres from
breeding to feeding grounds. In India, leatherback turtles nest mainly in the Andaman and Nicobar islands
beaches.
• Chelonia mydas (Green turtle)- This is the largest species found in the Indian waters. It occurs in the west
and east coasts of India, Lakshadweep and Andaman & Nicobar Islands. The name Green turtle indicates
the green colour of the fat. It is predominantly herbivorous and feeds on seagrass.
• Kemp’s ridley Turtle- Found primarily in the Gulf of Mexico, but also as far north as Nova Scotia, Kemp’s
ridleys are among the smallest sea turtles, reaching only about 2 feet in shell length and weighing up to 100
pounds.
• Hence option (d) is the correct answer.

Q 12.D
• Epidermis: The epidermis in plants is a layer of cells that usually covers the roots, stems, leaves and flowers
of plants. The epidermis is the outer layer of cells of all aboveground (i.e., aerial) organs that are in the
primary stage of growth. The epidermis persists throughout the life of leaves and flowers, because these
organs do not have secondary growth.
• In some plants living in very dry habitats, the epidermis may be thicker since protection against water
loss is critical. The entire surface of a plant has this outer covering of epidermis. It protects all the
part of the plant. Hence statements 1 and 2 are correct.
• Epidermal cells on the aerial parts of the plant often secrete a waxy, water-resistant layer on their
outer surface. This aids in protection against loss of water, mechanical injury and invasion by parasitic
fungi. Since it has a protective role to play, cells of epidermal tissue from a continuous layer without
intercellular spaces. Most epidermal cells are relatively flat. Often their outer and side walls are thicker
than the inner wall. Hence statement 3 is correct.

Q 13.A
• The Press Council of India is a statutory body created by the Press Council Act of 1978. It is the apex body
for the regulation of the Press in India. It enjoys independence from the government. Hence statement 1
is correct.
4 www.visionias.in ©Vision IAS
• It acts as the regulator that defines and discharges professional standards for the print media in India. Hence
statement 2 is correct.
• It is considered the most important body that sustains democracy and ensures that freedom of speech. The
objective of the Press Council of India is to preserve the freedom of the press and of maintaining and
improving the standards of the press in India. It arbitrates the complaints against and by the press for
violation of ethics and for violation of the freedom of the press respectively.
• The Council shall consist of a Chairman and twenty-eight other members. The Chairman shall be a person
nominated by a committee consisting of the Chairman of the Council of States (Rajya Sabha), the Speaker
of the House of the People (Lok Sabha), and a person elected by the members of the Press Council. Hence
statement 3 is not correct.

Q 14.C
• The Union Cabinet chaired by Prime Minister recently approved the proposal to set up the National
Recruitment Agency (NRA).
• At present, candidates seeking government jobs have to appear for separate examinations conducted by
multiple recruiting agencies for various posts. Candidates have to pay fee to multiple recruiting agencies
and also have to travel long distances for appearing in various exams.
• Approximately 1.25 lakh government jobs are advertised every year for which 2.5 crore aspirants appear in
various examinations. A common eligibility Test would enable these candidates to appear once and apply
to any or all of these recruitment agencies for the higher level of examination.
• The NRA will conduct the Common Eligibility Test (CET) for recruitment to non-gazetted posts in
government and public sector banks. This test aims to replace multiple examinations conducted by different
recruiting agencies for selection to government jobs advertised each year, with single online test. Hence,
statement 1 is correct.
• Salient features:
o The Common Eligibility Test will be held twice a year.
o There will be different CETs for graduate level, 12th Pass level and 10th pass level to facilitate
recruitment to vacancies at various levels.
o The CET will be conducted in 12 major Indian languages. This is a major change, as hitherto
examinations for recruitment to Central Government jobs were held only in English and Hindi. Hence,
statement 2 is not correct.
o To begin with CET will cover recruitments made by three agencies : viz. Staff Selection Commission,
Railway Recruitment Board and the Institute of Banking Personnel Selection. This will be expanded in
a phased manner.
o There will be an examination centre in every district of the country. There will be a special thrust on
creating examination infrastructure in the 117 aspirational districts.
o CET will be a first level test to shortlist candidates and the score will be valid for three years.
o There shall be no restriction on the number of attempts to be taken by a candidate to appear in the CET
subject to the upper age limit. Age relaxation for SC/ST and OBC candidates as per existing rules will
apply. Hence, statement 3 is correct.

Q 15.B
• The belt of sub-polar low pressure is located between 60°-65° latitudes in both the hemispheres, which is
at the border of the temperate and frigid zone.
• At subtropical high-pressure belt, the air gets divided into two parts – one part goes towards the equatorial
region and the other part goes towards poles. The part of the wind that goes towards poles collides with the
winds coming from poles, at 60°-65° N and S. But the winds collide nearer to the surface. When they collide
and converge nearer to the surface they do not have any place to sink, so they rise. And when air rise it is
associated with clouds and precipitation. It is creating a similar condition that exists in the skies of equatorial
region. That is how it creates a low pressure zone which is called as sub-polar low.
• The pattern of atmospheric circulation from sub-tropical region to sub-polar region is called Ferrel Cell.
• The air that rise at the sub-polar low, it gets further divided into two part – one goes towards extreme side
of poles and the other goes towards subtropic. And that’s how a chain or loop is created where the air keeps
moving from equator to poles and vice versa.
• Due to low temperatures the subpolar low-pressure belts are not very well pronounced year long.
• It is not thermally induced because there is low temperature throughout the year and as such there should
have been high pressure belt instead of low pressure belt. Hence, statement 1 is not correct.
• These are dynamically produced due to
o Coriolis Force
5 www.visionias.in ©Vision IAS
o Ascent of air as a result of convergence of westerlies (coming from the subtropical high-pressure
regions) and polar easterlies (coming from the polar regions).
• The sub polar low pressure belt is more developed and regular in the southern hemisphere (because of over
dominance of oceans) while it is broken in the northern hemisphere. Hence, statement 2 is correct.
• Instead of regular and continuous belt there are well defined low pressure centres or cells over the oceans
in the northern hemisphere. For example, in the neighbourhood of Aleutian Islands in the Pacific Ocean and
between Geenland and Iceland in the Atlantic Ocean. Hence, statement 3 is correct.

Q 16.D
• Goods & Services Tax Council is a constitutional body for making recommendations to the Union and
State Government on issues related to Goods and Service Tax (GST).
• As per Article 279A (4), The Goods and Services Tax Council shall make recommendations to the Union
and the States on
o the taxes, cesses and surcharges levied by the Union, the States and the local bodies which may
be subsumed in the goods and services tax (GST). Hence statement 1 is correct.
o the goods and services that may be subjected to, or exempted from the goods and services tax;
o The threshold limit of turnover below which goods and services may be exempted from goods and
services tax;
o special provision with respect to the States of Arunachal Pradesh, Assam, Manipur,
Meghalaya, Mizoram, Nagaland, Sikkim, Tripura, Himachal Pradesh and Uttarakhand
o any special rate or rates for a specified period, to raise additional resources during any natural
calamity or disaster.
§ Kerala government levied a 1 per cent cess on the value of goods and services a disaster relief
cess (Kerala flood) to raise additional revenue in 2019 after approval by the Goods and Services
Tax Council. Hence statement 3 is correct.
o It shall recommend the date on which the goods and services tax be levied on petroleum crude,
high-speed diesel, motor spirit (commonly known as petrol), natural gas and aviation turbine
fuel. Hence statement 2 is correct.
• Additional Information about GST council:
o It consists of the following members, namely:
§ Union Finance Minister: Chairperson;
§ Union Minister of State in charge of Revenue or Finance: Member;
§ Minister in charge of Finance or Taxation or any other Minister nominated by each State
Government: Members.
6 www.visionias.in ©Vision IAS
o One-half of the total number of Members of the GST Council constitutes the quorum at its
meetings.
o Every decision of the Council is taken at a meeting, by a majority of not less than three-fourths of
the weighted votes of the members present and voting
§ Weightage of vote of the Central Government = 1/3rd of total votes cast
§ Weightage of vote of all the State Governments taken together = 2/3rd total votes cast, in that
meeting.

Q 17.B
• Amir Khusrau (1253–1325) was a contemporary of Balban, Jalaluddin Khilji and Alauddin Khilji. He
introduced many new ragas such as Ghora and Sanam. He evolved a new style of light music known as
Qawwalis by blending the Hindu and Iranian systems. The invention of the sitar was also attributed to him.
• Credited to have introduced the ghazal style of the song into India, Amir khusro was also the one who
introduced Persian, Arabic and Turkish elements into Indian classical music. Through this fusion, he created
many ragas like zilf, maan, sajgari. Hence statement 1 is correct.
• Amir Khusrau wrote a masnavi on the Tughlaq dynasty ruler, Mubarak Shah as "Nuh Sipihr" (Nine Skies)
relating the events of Mubarak Shah's rule. Hence statement 2 is not correct.
• Amir Khusrau gave a unique form to the Chishti Sama by introducing the qaul, a hymn sung at the opening
or closing of qawwali. Qawwals at the shrine of Shaikh Nizamuddin Auliya started their recital with the
qaul. Hence Statement 3 is correct.

Q 18.D
• Prime Minister Narendra Modi inaugurated a 351-km section between Khurja and Bhaupur in Uttar Pradesh
for commercial operations of the Dedicated Freight Corridor (DFC). Freight trains plying on this section
from now on will help decongest the existing Kanpur-Delhi main line of Indian Railways, which currently
handles trains at 150% of its line capacity.
• Foodgrain and fertilisers from the northern region are transported to the eastern and Northeast regions. From
East and Northeast, coal, iron ore, jute and petroleum products are transported North and West.

• Around 70% of the freight trains currently running on the Indian Railway network are slated to shift to the
freight corridors, leaving the paths open for more passenger trains, those trains can, in turn,
achieve better punctuality. Hence, statement 1 is correct.
• Tracks on DFC are designed to carry heavier loads than most Indian Railways. DFC will get track access
charge from the parent Indian Railways, and also generate its own freight business. Hence, statement 3 is
correct.
• The existing industrial areas of Aligarh, Khuja, Firozabad, Agra and Bhaupur will become major growth
centres of the area. These areas are agriculture hubs producing potato, paddy and maize. The agricultural
produce thus may get a pan-India market because of cheaper and faster DFC connectivity. Hence,
statement 2 is correct.

Q 19.D
• Anti-dumping is a protectionist tariff that a domestic government imposes on foreign imports that it believes
are priced below fair market value. Dumping happens when a country exports an item at a price lower than
the price of the product in its domestic market. While the Ministry of Commerce and Industry recommends
7 www.visionias.in ©Vision IAS
the Anti- dumping duty, it is the Ministry of Finance, which levies such duty. Hence statement 1 is
correct.
• Countervailing duty is an additional import duty imposed on imported products (by the importing country)
when such products enjoy benefits like export subsidies and tax concessions in the country of their
origin. The countervailing measures in India are administered by the Directorate General of Anti-dumping
and Allied Duties (DGAD), Ministry of Commerce. Hence statement 2 is correct.
• WTO's Special Safeguard Mechanism (SSM) is a tool that will allow developing countries to raise tariffs
temporarily to deal with import surges or price falls related to agricultural imports that are causing
injuries to domestic farmers. Hence statement 3 is correct.

Q 20.D
• Sargasso Sea: This is an anticyclonic circulation of ocean currents comprising the north equatorial current,
the Gulf stream and the canary current in the north Atlantic ocean. The water confined in this gyral is calm
and motionless.
• Thus, the motionless sea of the said gyral is called Sargasso sea which is derived from the Portugese word
'sargassum' meaning thereby sea weeds.
• It is located in the transition zone of the trade winds (N.E.Trades) and the westerlies. Hence, statement
1 is correct.
• This zone is characterized by the subsidence of air from above and the resultant anticyclonic
conditions.Thus, the anticyclonic conditions cause atmospheric stability and hence there are very feeble
and calm winds due to which there is little mixing of confined water (sargasso sea) with the remaining
waters of the North Atlantic Ocean. Hence statement 2 is correct.
• It records the highest salinity of the Atlantic Ocean due to high temperature and evaporation. The
salinity is also increased because of no mixing of the water of the sargasso sea with the remaining water of
the North Atlantic Ocean. Hence, statement 3 is correct.

Q 21.B
• Sholas are the local name for patches of stunted tropical montane forest found in valleys amid rolling
grassland. Sholas are found in the upper reaches of the Nilgiris, Anamalais, Palni hills, Kalakadu,
Mundanthurai and Kanyakumari in the states of Tamil Nadu and Kerala. Hence statement 1 is not correct.
• The vegetation that grows in Shola forests is evergreen. The trees are stunted and have many branches.
Their rounded and dense canopies appear in different colours. As tree species of the montane, evergreen
forests are flammable, regeneration of any Shola tree species is completely prevented except for
Rhododendron nilagiricum, the only Shola tree that can tolerate fire. Hence statement 2 is correct.

Q 22.B
• The term Mahayana is a Sanskrit word that literally means Great Vehicle. It believes in the
heavenliness of Buddha and idol worship of Buddha and Bodhisattvas embodying Buddha
Nature. Mahayana is the major branch of Buddhism today which has the maximum number of
followers compared to other Buddhist sects. Mahayana Buddhism also refers to the path of the
Bodhisattva which seeks complete enlightenment for the benefit of all sentient beings, hence it is also known
as ‘Bodhisattvayāna’, or the ‘Bodhisattva Vehicle’. The Mahayana tradition is the largest major tradition of
Buddhism existing today. Mahayana is further divided into sects- Madhyamika and Yogacara.
• Hinayana: Literally Lesser vehicle, also known as Abandoned Vehicle or Defective vehicle. It believes
in the original teaching of Buddha or the Doctrine of elders.
8 www.visionias.in ©Vision IAS
• Hinayana does not believe in Idol worship and tries to attain individual salvation through self-
discipline and meditation. They believe in the original teachings of Buddha. Hence statement 1 is not
correct.
• Hinayana generally calls themselves Theravada (teaching of elders). It is the Buddha path for those who
wish to be Arhat (a perfect saint). Arhats are those saints who get enlightened and attain nirvana in a short
period of time. This branch of Buddhism is the second-largest school of Buddhism after
Mahayana. This school is found and followed in Srilanka, Cambodia, Laos, Burma, and Thailand because
of which Hinayana is also known as Southern Buddhism. Hinayana is further divided into two schools
known as – Vaibhasika and Sautrantika.
• Hinayana favoured the pali language whereas Mahayana favoured the Sanskrit language. Hence
statement 2 is correct.

Q 23.B
• There were at least three different kinds of people living in most villages in the southern and northern parts
of the subcontinent. In the Tamil region, large landowners were known as vellalar, ordinary
ploughmen were known as uzhavar, and landless labourers, including slaves, were known as kadaisiyar
and adimai.
• In the northern part of the country, the village headman was known as the grama bhojaka. The grama
bhojaka was often the largest landowner. Generally, he had slaves and hired workers to cultivate the land.
The king often used him to collect taxes from the village. He also functioned as a judge, and sometimes as
a policeman. Apart from the gramabhojaka, there were other independent farmers, known as grihapatis,
most of whom were smaller landowners. And then there were men and women such as the dasa
karmakara, who did not own land and had to earn working as labour.
• Hence option (b) is the correct answer.

Q 24.A
• Estuaries are located where the river meets the sea, they are the most productive water bodies in the
world. Estuaries experience the flow of fresh water from the rivers with salt water transported from the
tides. Hence statement 1 is correct.
• Estuaries have very little wave action, and so they provide a calm refuge from the open sea despite having
a free connection. It is the most productive region as it receives a high amount of nutrients from fresh and
marine water. Hence statement 3 is not correct.
• Most of India’s major estuaries occur on the east coast, in contrast, estuaries on the west coast of the
country are smaller. Two typical examples of estuaries on the west coast are the Mandovi and Zuari
estuaries. Hence statement 2 is not correct.
• Many different habitat types are found in and around estuaries, including shallow open waters, freshwater
and saltwater marshes, swamps, sandy beaches, mud and sand flats, rocky shores, oyster reefs, mangrove
forests, river deltas, tidal pools, and seagrass beds.

Q 25.D
• Netravati River:
o The Netravati River has its origins in the Chikkamagaluru district of Karnataka.
o This river flows through the famous pilgrimage place Dharmasthala and considered as one of the Holy
rivers of India.
o It merges with the Kumaradhara River at Uppinangady before flowing to the Arabian Sea.
o This river is the main source of water for Bantwal and Mangalore.
o Netravati river flows south of Mangalore city.
o Earlier it was known as the Bantawal river.
o Hence, pair 1 is correctly matched.
• Nag river:
o Recently, the Bombay High Court said that the Nag river has become extremely polluted due to
industrialisation and urbanisation.
o The Nagpur city derives its name from the Nag river which passes through the city.
o The Nag river originates from the Ambazari Lake in west Nagpur.
o Major Tributaries - Pioli river.
o Endpoint - confluence with Kanhan River.
o Kanhan river is a tributary of Wainganga river.
o Hence, pair 2 is correctly matched.

9 www.visionias.in ©Vision IAS


• Mahisagar river (Mahi):
o It originates from the northern slopes of Vindhyas in the Dhar district of Madhya Pradesh.
o It drains into the Arabian Sea through the Gulf of Khambhat.
o Vadodara is the only important urban centre in the basin.
o Mahi is one of the major interstate west flowing rivers of India
o Hence, pair 3 is correctly matched.

Q 26.C
• Recent Context: Natesa is a rare sandstone idol from the Pratihara Style of architecture in Rajasthan. It
was smuggled out of the country in 1998, to be returned to India recently by the UK.
• About Gurjara-Pratihara dynasty
o Gurjara-Pratihara dynasty ruled much of Northern India from the mid-8th to the 11th century.
o The Pratiharas derived their name from the Sanskrit meaning doorkeeper, are seen as a tribal group or
a clan of the Gurjaras.
o Nagabhata I, Nagabhata Il, Mhir Bhoj (Bhoja) and Mahendrapala Iwere the other important rulers of
the dynasty.
o The expansion of the Gurjara-Pratihara kingdom involved constant conflicts with other contemporary
powers such as the Palas and the Rashtrakutas known as the tripartite struggle over the city of
Kannauj.
• Architecture of Gurjara-Pratihara dynasty
o The Pratiharas extended Gupta architecture tradition by adding a full-fledged mandapa to the single
shrine temple.
o They were known for their open pavilion temples.
o They are known for their new space concepts, structural and functional constructs, motifs etc. together
with aesthetic and iconographic norms temples belonging to Nagara Style of temple Architecture.
o Primary literary sources include the accounts of Arab merchants like Suleiman, al-Masudi and poet
Rajashekhara in court of Mahendrapala I.
Q 27.A
• The Central Bureau of Investigation traces its origin to the Special Police Establishment (SPE) which was
set up in 1941 by the Government of India. The functions of the SPE then were to investigate cases of
bribery and corruption in transactions with the War & Supply Deptt. Of India during World War II.
• The CBI derives its powers to investigate from the Delhi Special Police Establishment (DSPE) Act, 1946
which grants the CBI power to investigate a case in Delhi, without any permission, since it is part of the
Centre. However, in all the states, the CBI needs the consent of the state to investigate any case relating to
that state or having jurisdiction of that state since law and order is a state subject. Hence, statement 1 is
correct.
• There are two types of consent for a probe by the CBI. These are: general and specific.
o When a state gives a general consent (Section 6 of the Delhi Special Police Establishment Act) to the
CBI for probing a case, the agency is not required to seek fresh permission every time it enters that state
in connection with investigation or for every case.
o When a general consent is withdrawn, CBI needs to seek case-wise consent for investigation from the
concerned state government. If specific consent is not granted, the CBI officials will not have the power
of police personnel when they enter that state.\
• This hurdle impedes seamless investigation by the CBI. A general consent is given to facilitate that seamless
investigation in a case of corruption or violence.
• The CBI can begin a probe under certain circumstances — if a state government makes a request and the
Union government agrees to it, if the Supreme Court or any High Court order the CBI to take up such
investigations, or if the state government issues a notification of consent under section 6 of the DSPE Act
and the Union government issues notification under section 5 of the DSPE Act for an investigation. Hence,
statement 2 is correct.
• The CBI can initiate suo motu investigations only in Union Territories, under section 2 of the DSPE Act.
• It will have no impact on investigation of cases already registered with CBI as old cases were registered
when general consent existed. Hence, statement 3 is not correct.

Q 28.A
• Visakadatta was a celebrated author of the Gupta period (the patron). He was the author of two Sanskrit
dramas, Mudrarakshasa and Devichandraguptam. The Mudrarakshasa was a drama in Sanskrit.
Although written during the Gupta period, it describes how Chandragupta with the assistance of Kautilya
overthrew the Nandas. Hence pair 1 is not correctly matched.
10 www.visionias.in ©Vision IAS
• Banabhatta was a 7th-century Sanskrit prose writer and poet. He was the court poet in the court of King
Harsha Vardhana. Bana's principal works include a biography of Harsha (or Harshavardhana of the
Pushyabhuti dynasty), the Harshacharita, and one of the world's earliest novels, Kadambari. Both these
works are noted texts of Sanskrit literature. Hence pair 2 is correctly matched.
• The Rashtrakutas widely patronized the Sanskrit literature and the Jain literature Amogavarsha I, who was
a Jain patronized many Jain scholars. His teacher Jinasena composed Parsvabhudaya, a biography of Parsva
in verses. Another scholar Gunabhadra wrote the Adipurana, the life stories of various Jain saints. Hence
pair 3 is not correctly matched.

Q 29.C
• The Central and Eastern Europe (CEE) member countries have recently accepted the “Sinatra Doctrine” as
a counter to increasing China’s aggressiveness to undermine the unity of the European Union through the
divide and rule policy.
• The Soviet government of Mikhail Gorbachev used to describe their policy as Sinatra Doctrine. It allowed
the neighboring Warsaw Pact states to determine their internal affairs. The name was taken from the song
“My Way” which was popularized by Frank Sinatra. The implementation of the doctrine was part of the
doctrine of new political thinking by Gorbachev. Hence, option (c) is the correct answer.
• The Sinatra Doctrine will be based on two pillars:
o Continuing cooperation with China with respect to address global challenges like covid-19, climate
change & regional conflicts and
o Strengthening the strategic sovereignty of the European Union by protecting the technological sectors
of its economy
• Chinese policy to undermine European unity was fuelled by leveraging the regional platform in order to get
political favors in the exchange for economic benefits.
• The credit-based offer of China in a neo-colonial fashion was not suitable for CEE members of EU. Further,
the Chinese investments in 12 EU member states which were participating in “17+1 initiative” was
approximately 8.6 billion euros in the year 2010 to 2019. On the other hand, the Chinese investment in
Finland for the same period was 12 billion euros and in the Netherlands, it was 10.2 billion euros. Thus, this
mismatch between the economic promises and outcome by Beijing, made the CEE members to adopt the
Sinatra doctrine.
o 17+1 initiative
§ It is also known as the “Cooperation between China and Central and Eastern European Countries”.
§ It is an initiative of China to promote business and investment relations between China and
members of CEE.
• The Central and Eastern Europe (CEE) member countries include- Bosnia, Albania, Herzegovina, Croatia,
Bulgaria, Estonia, the Czech Republic, Hungary, Greece, Latvia, Lithuania, Montenegro, North Macedonia,
Romania, Poland, Slovakia, Serbia and Slovenia.
Q 30.A
• Recent Context: The National Centre for Polar and Ocean Research (NCPOR) has predicted an increase
in the expansion of Amery Ice Shelf boundaries from its 2016 positions. It has been predicted that there
would be a 24% increase in the expansion of AIS boundaries by 2021 and another 24% expansion by 2026
from its 2016 positions.
• Amery Ice Shelf is one of the largest glacier drainage basins in the world, located on the east coast of
Antarctica.
• The shelf dynamics and mass balance help in understanding the changes in the global climate scenario.
• The prediction made by NCPOR is expected to help to better understand the ongoing changes in the ocean
and atmospheric forces.
• It was also observed that the critical cooling of the Sea Surface Temperature (SST) is resulting in an
advancement of the ice shelf by 88% in the past 15 years. These changes would contribute in a major way
to climate variability.
• Ice Shelves: The floating sheets of ice are called ‘ice shelves. It connects a glacier to the landmass. It plays
a significant role in maintaining the stability of a glacier. The following are the important parameters for
the balancing of a glacier:
o Mass Balance: The ice sheet mass balance of a glacier is the net balance between the mass gained by
snow deposition and the loss of mass by melting.
o Sea stratification: It occurs when water masses with different properties - salinity, oxygenation, density,
temperature - form layers that act as barriers to water mixing.
o Deep-ocean bottom water formation: It is formed by the cooling and sinking of surface water at high
latitudes.
11 www.visionias.in ©Vision IAS
Q 31.D
• Rash Behari Bose was an Indian revolutionary leader who fought tooth and nail against the British Raj.
Born in Bengal, he left Bengal to shun the Alipore bomb case trials of (1908). He played a key role in
the Delhi Conspiracy Case(attempted assassination on the British Viceroy, Lord Hardinge). On
December 23, 1912, a bomb was thrown at Lord Hardinge during a ceremonial procession of transferring
the capital from Calcutta to New Delhi. Bose was the mastermind behind the attack. After it was failed,
Bose escaped while four others were hanged.
• During World War I, he became extensively involved as one of the leading figures of the Ghadar
Revolution that attempted to trigger a mutiny in India in February 1915. The British, however, foiled all
attempts of Bose to create unrest and arrested several of Bose's confidants; of which twenty-eight were
hanged after trials under what came to be known as the Lahore Conspiracy Case. Bose managed to escape
from India in 1915 and went to Japan where he lived as a fugitive.
• In 1942, Bose established the ‘Indian Independence League’ — an army that would fight for India’s
freedom from British rule. He subsequently handed over the reins to Subhas Chandra Bose, who developed
it as the Indian National Army or the Azad Hind Fauj.
• Hence option (d) is the correct answer.

Q 32.C
• The Government of India Act 1935 proposed for an All India Federation of British India, Princely
states, and the Commissioners provinces. The federation though never came up due to the non-fulfilment
of two conditions of the act that were:
o states with the allotment of 52 seats in the proposed council of states should agree to join the federation.
o the aggregate population of states in the above category should be 50% of the total population of all
Indian states.
• The bicameral legislature proposed by the act was to have an upper house that is the council of states and
the lower house that is the federal house. Oddly enough the elections to the Council of states were to be
direct from the British Indian Provinces and partial representation of the princely states was through
nominations. Hence, statement 1 is not correct.
• The women were given the right to vote in the provincial legislature, thus expanding the franchise. The
members of the provincial legislature were to be directly elected. Hence statement 2 is correct.
• Among the principal sources from which the Government of India Act, 1935 drew its material were:
o Indian Statutory Commission Report (popularly known as Simon Commission report),
o The report of the All Parties Conference (the Nehru Report),
o A white paper issued after the discussions at the three round table conferences in 1933, which
gave details of the working basis of the new constitution of India i.e. dyarchy at the Centre and
responsible government.
o The Joint select committee report and
o The Lothian report, which determined the electoral provisions of the act. Hence statement 3 is correct.

Q 33.A
• Akbar made some experiments in the land revenue administration with the help of Raja Todar Mal. The
land revenue system of Akbar was called Zabti or Bandobast system. It was further improved by Raja
Todar Mal. It was known as Dahsala System which was completed in 1580.
• By this system, Todar Mal introduced a uniform system of land measurement. The revenue was fixed on
the average yield of land assessed on the basis of the past ten years. Hence statement 1 is correct.
• The main features of the Zabti system as it finally came into operation under Akbar were:
o Measurement of land was essential
o Fixed Cash revenue rates are known as Dastur for each crop
o Due to fixed Dasturs, local officials could not use their discretion. Hence statement 2 is correct.
o One-third of the average produce was the state's minimum share. Hence statement 3 is not correct.
o With fixing the permanent Dasturs, the uncertainties and fluctuations in levying the land revenue
demand were greatly reduced
• The land was also divided into four categories – Polaj (cultivated every year), Parauti (once in two years),
Chachar (once in three or four years) and Banjar (once in five or more years). Payment of revenue was made
generally in cash.

12 www.visionias.in ©Vision IAS


Q 34.B
• Hornbills are large and wide-ranging birds. India has nine hornbill species, of which four are found in the
Western Ghats: Indian Grey Hornbill (endemic to India), the Malabar Grey Hornbill (endemic to the
Western Ghats), Malabar Pied Hornbill (endemic to India and Sri Lanka). Great hornbills are found in the
forests of India, Bhutan, Nepal, Mainland Southeast Asia, the Indonesian Island of Sumatra, and the
Northeastern region of India. Hence statement 1 is not correct.
• Great Hornbills Buceros bicornis, among the largest birds in Asian tropical rainforests, depend on large
trees for nesting and a diverse array of patchily distributed fruiting trees. The nesting cycle ranges from
seven to eight nests varied between 114 and 130 days. For this purpose, it needs large trees for both food
requirements and protection. Hence statement 2 is correct.
• Great hornbill calls are loud and aggressive, especially to send an alarm to the flock when they spot birds
of prey. It's a reflex behavior that has been nurtured in great hornbill species owing to their gregarious nature
and protective parenting. Hence statement 3 is correct.

Q 35.C
• An extreme helium star or EHe is a low-mass supergiant that is almost devoid of hydrogen, the most
common chemical element of the universe. There are 21 of them detected so far in our galaxy. Hence
statement 1 is correct.
• Their severe chemical peculiarities challenge the theory of well-accepted stellar evolution as the observed
chemical composition of these stars do not match with that predicted for low mass evolved stars.
• Extreme helium stars (EHes) are supergiants much larger and hotter than the sun, but less massive.
EHes are characterized by relatively sharp and strong lines of neutral helium, which indicates low
surface gravities and atmospheres dominated by helium. Besides helium, these stars also have significant
amounts of carbon, nitrogen and oxygen. EHe was first detected in 1942. Hence statement 2 is correct.

Q 36.B
• Glanders is a highly infectious and often fatal zoonotic disease primarily affecting horses, donkeys and
mules. It is a notifiable disease of equines caused by Gram-negative bacterium, Burkholderia mallei. It is a
rare disease in humans with cases having occurred in veterinarians, other people working closely with
horses, and laboratory workers handling the organism.
• Despite many reported cases of the disease in equines, no human case of glanders has been reported in India
till date.
• Hence option (b) is the correct answer.

Q 37.C
• ‘Gause's competitive exclusion principle (CEP) states that two species with identical niches cannot coexist
indefinitely as they compete for identical resources. A species' niche is basically its ecological role, which
is defined by the set of conditions, resources, and interactions it needs. Hence Statement 1 is correct.
• In nature, it's rarely the case that two species occupy exactly identical niches. However, the greater the
extent to which two species' niches overlap, the stronger the competition between them will tend to
be. Hence statement 2 is correct.
• Competitive exclusion may be avoided if one or both of the competing species evolves to use a different
resource, occupy a different area of the habitat, or feed during a different time of day. The result of
this kind of evolution is that two similar species use largely non-overlapping resources and thus have
different niches. This is called resource partitioning, and it helps the species coexist because there is less
direct competition between them.

Q 38.D
• India’s External Debt to GDP ratio rose marginally to 21.6% (end-September 2020) compared to 20.6%
(end-March 2020) - however there is no consistent trend - 2019 (19.8), 2018 (20.1), 2013 (22.4). Hence,
statement 1 is not correct.
• Debt Service Ratio which refers to principal payment plus interest payment has improved to 9.7% (end-
September 2020) compared to 6.5% (end-March 2020). Hence, statement 2 is not correct.
• Refer the table below:

13 www.visionias.in ©Vision IAS


Q 39.A
• The wildcat has long legs, a short face, long canine teeth, and distinctive ears — long and pointy, with tufts
of black hair at their tips. The iconic ears are what give the animal its name — caracal comes from the
Turkish karakulak, meaning ‘black ears’. In India, it is called Siya gosh, a Persian name that translates as
‘black Ear’. Hence statement 1 is correct.
• The caracal (Caracal caracal) is a medium-sized wild cat. The National Board for Wildlife and Union
Ministry of Environment, Forest and Climate Change last month included the caracal, a medium-sized
wildcat found in parts of Rajasthan, Madhya Bharat pathar and Gujarat, in the list of critically endangered
species. However globally it is in the least concerned (Lc) category of the IUCN Red List. Hence statement
2 is not correct.
• Caracal' naturally feeds on small livestock like the baby of goat, hen, and several amphibians and reptiles
near river banks. Chambal was once used to be the hotspot of Caracal however due to the encroachment of
ravines of Chambal and its declaration of wasteland their population dwindled in India. Hence statement
3 is correct.

Q 40.A
• Mahatma Gandhi and Untouchability
o Gandhiji believed that standing at the heart of the inherited Hindu tradition, including its caste system,
it was possible to overcome untouchability. He said, "In my opinion, untouchability is a blot on
humanity and therefore upon Hinduism. It cannot stand the test of reason. It is in conflict with the
fundamental precepts of Hinduism,” he insisted. Untouchability was reform “custom masquerading
under the name of religion.” He set out to reform but not to reject Hinduism. According to the Mahatma,
"the caste system is a hindrance, not a sin. But untouchability is a sin, a great crime, and if Hinduism
does not destroy this serpent while there is time, it will be devoured by it." He firmly believed that
ultimately the removal of untouchability depended on the change of heart of millions of caste Hindus.
o Gandhiji was not in favour of mixing up the issue of removal of untouchability with that of inter-
caste marriages and with that of inter-dining because he felt that such restrictions were very much a
part of Hindus as well as the Harijans too. Hence statement 2 is not correct. Gandhiji distinguished
between the abolition of untouchability and the abolition of the caste system as such. He believed that
the removal of untouchability would have a positive impact on communal and other questions since
opposition to untouchability meant opposing the notion of highness and lowness.
o The Belgaum (now Belagavi) Congress session was a momentous event as it was the only session
chaired by Gandhiji. His primary objective was to restore unity between Hindus and Muslims,
remove untouchability, besides underlying the importance of charkha and khadi. Hence statement 1
is correct.

Q 41.A
• Aestivation is also known as summer sleep. it is defined as the inactivity of the organisms during the summer
season in order to reduce their loss of energy through increased metabolic rate.
• It also helps to prevent desiccation from the effects of the high temperature in tropical regions. Many
reptiles, amphibians, insects, and even mammals practice aestivation. Eg. The Madagascan fat-tailed dwarf
lemur, Crocodiles, Salamanders, Frogs, earthworms, etc. However snakes and bear practices hibernation
that is winter sleep. Hence option (a) is the correct answer.
14 www.visionias.in ©Vision IAS
Q 42.A
• Recent Context: Permanent Indus Commission recently met after a gap of more than two-and-a-half years,
a period that witnessed: Pulwama attack (14th February 2019), Balakot airstrike (26th February 2019), and
Abrogation of special provisions under Article 370 that gave special status to J&K.
• Permanent Indus Commission
o It is a bilateral commission of officials from India and Pakistan, created to implement and manage goals
of the Indus Waters Treaty, 1960 brokered by the World Bank. Hence, statement 1 is correct and 2 is
not correct.
o The Commission, according to the treaty, shall meet regularly at least once a year, alternately in India
and Pakistan.
o The functions of the Commission include:
§ to study and report to the two Governments on any problem relating to the development on the
waters of the rivers.
§ to solve disputes arising over water sharing.
§ to take necessary steps for the implementation of the provisions of the treaty.
• Indus Water Treaty, 1960:
o It is a treaty brokered by the World Bank and signed by then Prime Minister Jawaharlal Nehru and
Pakistan’s President Ayub Khan which administers how the waters of the Indus and its tributaries that
flow in both the countries will be utilized.
o According to the treaty, waters of the eastern rivers — Sutlej, Beas and Ravi had been allocated to India,
while the western rivers — the Indus, Jhelum and Chenab to Pakistan.
o Under the treaty, India has been given the right to generate hydroelectricity through a run of the river
projects on the western rivers subject to specific criteria for design and operation.
o It also gives Pakistan the right to raise concerns on the design of Indian hydroelectric projects on western
rivers.
o The Treaty also provides an arbitration mechanism to solve disputes amicably.
• World Bank, at multiple instances, has tried to seek an amicable resolution to the various disagreement and
protect the Treaty.

Q 43.B
• On-tap issue: This is a reissue of existing Government securities having pre-determined yields/prices by the
Reserve Bank of India. After the initial primary auction of a security, the issue remains open to further
subscription by the investors as and when considered appropriate by RBI. The period for which the issue is
kept open may be time specific or volume specific.
• The coupon rate, the interest rates and the date of maturity remain the same as determined in the initial
primary auction. Reserve Bank of India may sell government securities through on tap issue at lower or
higher prices than the prevailing market prices. Such an action on the part of the Reserve Bank of India
leads to a realignment of the market prices of government securities. Tap stock provides an opportunity to
unsuccessful bidders in auctions to acquire the security at the market-determined rate. Hence option (b) is
the correct answer.
Q 44.D
• The idea for the AIWC emerged in 1926, at the suggestion of Irish-born theosophist and feminist
Margaret Cousins. Nearly a decade earlier, in 1917, Cousins had helped establish the Women’s Indian
Association in Madras (now Chennai), one of India’s first feminist groups. Multiple such associations were
subsequently established in India. In 1926, concerned about issues in women’s education in India, Cousins
wrote an appeal to members of women’s associations across the country, calling on them to meet and outline
their thoughts regarding educational reform and to select a representative to attend a special conference
in Poona (Pune) and in 1927, The All India Women's Conference (AIWC) was founded in Pune by
Margaret Cousins. Hence statement 1 is correct.
• It aimed to improve educational efforts for women and children and has expanded its scope to also tackle
other women's rights issues. Hence statement 2 is correct.
• In January 1927 the delegates from the regional meetings gathered in Poona at the first official meeting of
the AIWC. The initial conference and the participation of women from a range of communities and
backgrounds signified an important chapter in the development of the women’s movement in India. At the
first conference, a number of resolutions were adopted, including those aimed at enriching the content
of primary education, vocational training, and collegiate-level programs.
• Amrit Kaur was one of the founding members of AIWC. Kamaladevi Chattopadhyay was an Indian social
reformer and freedom activist. She has been the driving force behind the upliftment of the socio-economic
standard of Indian women. She has also served as a secretary to the AIWC. Hence statement 3 is correct.
15 www.visionias.in ©Vision IAS
Q 45.A
• Digital services tax (DST) was introduced by the Indian government vide the 2020 Finance Act. The tax
applies only to non-resident companies with annual revenues in excess of ₹2 crores and covers online
sales of goods and services to Indians. Hence, statement 1 is correct.
• The DST is aimed at ensuring that non-resident, digital service providers pay their fair share of tax on
revenues generated in the Indian digital market. India’s 2% DST is levied on revenues generated from digital
services offered in India, including digital platform services, digital content sales, and data-related services.
• India was one of the first countries in the world to introduce a 6 per cent equalisation levy in 2016, but the
levy was restricted to online advertisement services (commonly known as “digital advertising taxes” or
“DATs”). The 2020 DST, however, is broader in scope and extends to all kinds of digital transactions.
• In a bid to provide a level-playing field, the government has decided not to levy 2% digital service tax
if goods and services are sold through an Indian arm of foreign e-commerce players. The amendment
to Finance Bill 2021 clarifies that offshore e-commerce platforms don’t have to pay the 2% equalisation
levy if they have a permanent establishment or they pay any income tax here. Hence, statement 2 is not
correct.

Q 46.C
• Burzahom
o The Neolithic Site of Burzahom, in the district of Srinagar, India brings to light transitions in human
habitation patterns from Neolithic Period to the Megalithic period to the Early Historic period. From
transition in architecture to development in tool-making techniques to introduction and diffusion of
lentil in north-western India, the site of Burzahom is a unique comprehensive story teller of life between
3000 BCE to 1000 BCE. Hence pair 3 is correctly matched.
o The remains of the site document the gradual change in the nature of dwelling spaces among early
societies. From subterranean dwelling pits, the evidence in the site shows the emergence of mud
structures, thereon mud-bricks constructions on level ground. The range of tools recovered from the site
shows the evolution in tool making Neolithic men skilled hunters and their knowledge in applying the
implements for cultivation.
• Bagor
o It is a large village on the left bank of the Kothari river, a tributary of river Banas, situated 25 kilometres
to the west of the town Bhilwara (Rajasthan). Bagor is the largest Mesolithic site (and not
megalithic) discovered in India and it is the only one to have been excavated horizontally which expose
extensive living floors. The site gives insight into the process of acculturation arising out of coming in
contact with full-fledged farming cultures. Hence pair 2 is not correctly matched.
• Gufkral (Kashmir)
o It is an aceramic neolithic site in the Kashmir Valley. Gufkral (literally guf-cave, kral-potter)-a site
inhabited by potters who utilize the caves cut into the karewa-was excavated by the Prehistory Branch
of the Archaeological Survey of India from 18 August to 20 October 1981. On the slopes of the 35 m
high mound, there are a number of caves, both single and multichambered with pillars. Some,
particularly on the southeastern side, are occupied by Krals both for residential and storage
purposes. Hence pair 1 is not correctly matched.

Q 47.B
• The third phase of the Indian Beamline project, a facility for materials research set up under India-Japan
Scientific and Technological Cooperation, was initiated on March 23, 2021, with special focus on
industrial application research. In the first phase (2009-2015) of this project, an X-ray beamline (BL18B)
was constructed by Saha Institute of Nuclear Physics in Japan, and over the years, this facility has
provided substantial amount of quality beamtime for the use of Indian scientists to carry out front-ranking
research in advanced materials including nanomaterials. Hence statement 2 is correct.
• The phase would increase the number of young researchers from India to be trained in advanced X-ray
techniques of material research. Besides, steps will be taken to allocate more beamtime so that more
researchers can get access to it.
• The Indian beamline has been constructed and maintained by Saha Institute of Nuclear Physics (SINP),
Kolkata and Jawaharlal Nehru Centre for Advanced Scientific Research (JNCASR), Bangalore, in
Japanese synchrotron light source Photon Factory (PF) of High Energy Accelerator Research
Organization (KEK), with support from NanoMission, Department of Science and Technology
(DST). Hence statement 1 and 3 are correct.

16 www.visionias.in ©Vision IAS


Q 48.D
• Additive manufacturing (AM), also known as 3D printing, is a transformative approach to industrial
production that enables the creation of lighter, stronger parts and systems.
• Additive manufacturing uses data computer-aided-design (CAD) software or 3D object scanners to direct
hardware to deposit material, layer upon layer, in precise geometric shapes. As its name implies, additive
manufacturing adds material to create an object. By contrast, when you create an object by traditional
means, it is often necessary to remove material through milling, machining, carving, shaping or other means.
• How does additive manufacturing work? The term “additive manufacturing” references technologies that
grow three-dimensional objects one superfine layer at a time. Each successive layer bonds to the preceding
layer of melted or partially melted material. Objects are digitally defined by computer-aided-design (CAD)
software that is used to create .stl files that essentially "slice" the object into ultra-thin layers. This
information guides the path of a nozzle or print head as it precisely deposits material upon the preceding
layer. Or, a laser or electron beam selectively melts or partially melts in a bed of powdered material. As
materials cool or are cured, they fuse together to form a three-dimensional object.
• Additive manufacturing technologies Sintering: Sintering is the process of creating a solid mass using
heat without liquefying it. Sintering is similar to traditional 2D photocopying, where toner is selectively
melted to form an image on paper.
• Direct Metal Laser Sintering (DMLS): Within DMLS, a laser sinters each layer of metal powder so that
the metal particles adhere to one another. DMLS machines produce high-resolution objects with desirable
surface features and required mechanical properties. With SLS, a laser sinters thermoplastic powders to
cause particles to adhere to one another.
• Direct Metal Laser Melting (DMLM) and Electron Beam Melting (EBM): By contrast, materials are
fully melted in the DMLM and EBM processes. With DMLM, a laser completely melts each layer of metal
powder while EBM uses high-power electron beams to melt the metal powder. Both technologies are ideal
for manufacturing dense, non-porous objects.
• Stereolithography (SLA): Stereolithography (SLA) uses photopolymerization to print ceramic objects.
The process employs a UV laser selectively fired into a vat of photopolymer resin. The UV-curable resins
produce torque-resistant parts that can withstand extreme temperatures.
• Additive manufacturing materials: It is possible to use many different materials to create 3D-printed
objects. AM technology fabricates jet engine parts from advanced metal alloys, and it also creates chocolate
treats and other food items.
• Thermoplastics: Thermoplastic polymers remain the most popular class of additive manufacturing
materials. Acrylonitrile butadiene styrene (ABS), polylactic acid (PLA) and polycarbonate (PC) each offer
distinct advantages in different applications. Water-soluble polyvinyl alcohol (PVA) is typically used to
create temporary support structures, which are later dissolved away.
• Metals: Many different metals and metal alloys are used in additive manufacturing, from precious metals
like gold and silver to strategic metals like stainless steel and titanium.
• Ceramics: A variety of ceramics have also been used in additive manufacturing, including zirconia,
alumina and tricalcium phosphate. Also, alternate layers of powdered glass and adhesive are baked together
to create entirely new classes of glass products.
• Biochemicals: Biochemical healthcare applications include the use of hardened material from silicon,
calcium phosphate and zinc to support bone structures as new bone growth occurs. Researchers are also
exploring the use of bio-inks fabricated from stem cells to form everything from blood vessels to bladders
and beyond.
• Additive manufacturing applications Aerospace: AM excels at producing parts with weight-saving,
complex geometric designs. Therefore, it is often the perfect solution for creating light, strong aerospace
parts.
• Automotive: Recently it was reported that the McLaren racing team is using 3D-printed parts in its
Formula 1 race cars. In the auto industry, AM's rapid prototyping potential garners serious interest as
production parts are appearing. For example, aluminium alloys are used to produce exhaust pipes and pump
parts, and polymers are used to produce bumpers.
• Healthcare: Additive manufacturing technology can be used to create custom, on-demand 3D printed
surgical implants for patients suffering from bone cancer. In general, healthcare applications for additive
manufacturing are expanding, particularly as the safety and efficacy of AM-built medical devices is
established. The fabrication of one-of-a-kind synthetic organs also shows promise. Recently, researchers
at Northwestern University in Chicago created the first artificial mouse ovaries using a 3D printer.
• 3D Printing and the Environment: Additive manufacturing is sometimes seen as inherently
environmentally preferable to conventional manufacturing because of its potential for local production —
by consumers, retailers and hobbyists — and because it is thought to allow zero-waste manufacturing.
17 www.visionias.in ©Vision IAS
Research in this issue, however, indicates that the environmental performance is very sensitive to the pattern
of usage and configuration of the machinery and the materials used.

Q 49.C
• Viceroy Canning had introduced the Portfolio system in 1859 that divided Government into several
branches, which were entrusted to different members of the Governor General's council. It also envisages
that the member in-charge of his department could issue final orders with regard to matters which concerned
his department. It laid the foundations of cabinet government in India. It was given legal status
under Indian Council Act of 1861. Hence statement 1 is correct.
• The Quit India Movement also known as the August Revolution, was a movement launched at the
Bombay session of the Indian National Congress by Mahatma Gandhi on 9 August 1942, during World
War II, demanding an end to British rule in India. After the failure of the Cripps Mission to secure Indian
support for the British war effort, Gandhi made a call to Do or Die in his Quit India speech delivered in
Bombay on 8 August 1942 at the Gowalia Tank Maidan. Lord Linlithgow was the Viceroy at that
time. Hence statement 3 is correct.
• Dadabhai Naoroji is also known as the "Grand Old Man of India". In 1867 he helped to establish the East
India Association. He was also a Liberal Party member of Parliament (MP) in the United Kingdom House
of Commons between 1892 and 1895 and the first Indian to be a British MP. His book 'Poverty and Un-
British Rule in India' brought attention to his theory of the Indian "wealth drain" into Britain.
• The Butler Committee (1927) was set up to examine the nature of relationship between the princely states
and government. It was appointed during the tenure of Lord Irwin. It recommended that States should not
be handed over to an Indian Government in British India, responsible to an Indian legislature, without the
consent of states. Hence statement 2 is not correct.

Q 50.D
• N.M. Joshi initially was a member of Gokhale’s Servants of India Society. He founded the Social Service
League at Bombay in 1911 with the aim of securing for the masses better and reasonable conditions of life
and work. He also founded the All India Trade Union Congress in 1920 at Bombay but left it in 1929 when
it showed leaning towards the Soviet Union and started the Indian Trades Union Federation. Hence pair 1
is correctly matched.
• H.N. Kunzru: He founded the Seva Samiti at Allahabad in 1914 with the objective of organising social
service during natural calamities, and promoting education, sanitation, physical culture, etc. He was also a
member of the Servants of India Society earlier. Hence pair 2 is correctly matched.
• Gopal Ganesh Agarkar: One of the greatest rationalist thinkers from Maharashtra, he advocated the power
of human reason. He was totally opposed to any blind dependence on tradition or false glorification of
India’s past. He founded the Deccan Education Society at Poona in 1884 in association with B.G. Tilak,
V.K. Chiplunkar and Madhavrao Namjoshi. Hence pair 3 is correctly matched.

Q 51.C
• Provision for setting up administration tribunals at the union and state levels has been made in part- XIV-
A of the constitution by 42nd Amendment Act, 1976.
• Art. 323-A authorizes Parliament to establish administrative tribunals to resolve disputes and complaints
regarding the service commission of persons employed under the union/state governments. The Parliament
provides for the jurisdiction, power, and authority to such tribunals and prescribes the procedure to be
followed by them. Art.323 B deals with tribunals for matters other than in Art.323 A. Hence statement 1
is not correct.
• Central Administrative Tribunal (CAT) set up under Article 323A, exercises original jurisdiction in relation
to recruitment and all service matters of public servants covered by it. Hence statement 2 is correct.
• The CAT can adjudicate on cases involving members in the government services except those in defence
forces, officers and servants of the Supreme Court and the secretarial staff of the Parliament. Hence
statement 3 is not correct.
• The CAT is not bound by the procedure laid down in the Civil Procedure Code of 1908. It is guided by the
principles of natural justice.

Q 52.D
• Kandukuri Veeresalingam Pantulu (1848 – 1919) was a social reformer and writer from the Madras
Presidency. He is considered the father of the Telugu Renaissance movement. He was one of the early
social reformers who encouraged the education of women and the remarriage of widows (which was not
supported by society during his time). Hence statement 1 is not correct.
18 www.visionias.in ©Vision IAS
• D.K. Karve or Dhondo Keshav Karve (1858–1962), popularly known as Maharshi Karve, was a social
reformer from the Bombay Presidency.
• Vishnu Shastri Pandit founded the Widow Remarriage Association in the 1850s. Another prominent worker
in this field was Karsondas Mulji who started the Satya Prakash in Gujarati in 1852 to
advocate widow remarriage.
• Similar efforts were made by Professor D.K. Karve in western India and by Veerasalingam Pantulu in
Madras. Karve himself married a widow in 1893. He dedicated his life to the upliftment of Hindu widows
and became the secretary of the Widow Remarriage Association. He opened a widows’ home in Poona to
give the high caste widows an interest in life by providing them with facilities for vocational training. Hence
statement 2 is not correct.

Q 53.D
• Black-backed woodpeckers - are perhaps the most iconic of post-fire species, since they’re wholly reliant
on burn zones. They forage at smaller, charred trees, but when it comes to finding a home, they single out
trees with previously decayed heartwood—nicknamed “spike tops”—where they excavate their nests the
year after a fire.
• Kerner blue butterfly caterpillar - depends on fire for its food source. Wild Lupine which is the sole food
source for butterflies requires fire to flower and flourish in the wildland of California USA.
• Leadbeater’s possum - too benefited from the fire regime, bushfires allowing time to naturally create
hollows in big trees. This hollowness helps in the creation of nests and safe homes for this species in trees
for the purpose of fertilization and rearing of offsprings. Hence option (d) is the correct answer.

Q 54.D
• Recent Context: The Federal Helium Reserve which is the major helium reserve of the USA is set to close
down production in 2021, and scientists are looking for new reserves to replace it. India imports helium for
its needs and in which the major chunk is from the USA but as it is appearing set to cut off exports of helium
since 2021, the Indian industry stands to lose out heavily. Hence statement 2 is not correct
• Indian Reserves:
o India’s Rajmahal volcanic basin in Jharkhand is the storehouse of helium trapped for billions of years,
since the very birth of Earth from the Sun. Hence statement 1 is not correct.
o At present, researchers are mapping the Rajmahal basin extensively for future exploration and
harnessing of helium.
• Need for India:
o To Reduce Import Burden:
o Every year, India imports helium worth Rs. 55,000 crores from the USA to meet its needs.
• Use:
o Helium is used in medicine, scientific research, for blimp inflation, party balloons as well as for welding
applications. It finds many applications, mainly in magnetic resonance imaging (MRI) scans, in rockets
and in nuclear reactors.
• USA’s Monopoly:
o After discovering that helium was concentrated in large quantities under the American Great Plains.
The USA became the most important exporter of helium across the world. It was soon realized that the
USA was also the biggest storehouse of helium.
Q 55.B
• Ultraviolet germicidal irradiation (UVGI) is an established means of disinfection and can be used to prevent
the spread of certain infectious diseases. UV-C radiation kills or inactivates microbes by damaging their
deoxyribonucleic acid (DNA).
• The principal mode of inactivation occurs when the absorption of a photon forms pyrimidine dimers
between adjacent thymine bases and renders the microbe incapable of replicating. UVGI can be used to
disinfect air, water, and surfaces. Hence statement 1 is correct.
• Low-pressure mercury vapor has a strong emission line at 254 nm, which is within the range of wavelengths
that demonstrate strong disinfection effect. Hence statement 2 is correct.
• UVC radiation is a known disinfectant for air, water, and nonporous surfaces. UVC radiation has effectively
been used for decades to reduce the spread of bacteria, such as tuberculosis. UVC radiation has been shown
to destroy the outer protein coating of the virus. However direct exposure of UVC radiation to virus is
required.
• Hence statement 3 is not correct.

19 www.visionias.in ©Vision IAS


Q 56.C
• Humayun who was a great patron of arts acquired the services of two main painters Abdus Samad
and Mir Sayyid Ali. These artists were responsible for bringing Persian influence in the Mughal paintings.
They created am illustrated manuscript during the reign of Akbar known as Tutinama. Hence statement 1
is not correct.
• The artist during the reign of Akbar encouraged the use of calligraphy in the paintings. One of the
distinguishing features of this period was the transformation of popular art to the court art that is the artist
was more focused at depicting the scenes of court life than the life of masses. Hence statement 2 is correct.
• Jahangir made the mughal painting reach its zenith, he was a naturalist and preferred flora and fauna
in paintings, while Shah Jahan tried to reduce the liveliness of the paintings due to the European
influence in his reign and personality. Shah Jahan liked to create artificial elements in the paintings and
ordered to increase the use of gold and silver in the paintings. Hence statement 3 is not correct.

Q 57.D
• A team of researchers from the Indian Institute of Science (IISc) and Indian Space Research Organisation
(ISRO) has jointly developed a sustainable process for making brick-like structures on the moon.
• It exploits lunar soil and uses bacteria and guar beans to consolidate the soil into possible load-bearing
structures. These “space bricks” could eventually be used to assemble structures for habitation on the
moon’s surface. Since the manufacturing process is organic and sustainable it reduces carbon footprint and
also reduces the expenditure cost of carrying the bricks from the Earth to the moon.
• Hence option (d) is the correct answer.

Q 58.C
• Political parties are voluntary associations or organised groups of individuals who share the same political
views and who try to gain political power through constitutional means and who desire to work for
promoting the national interest.
• The Election Commission registers political parties for the purpose of elections and grants them recognition
as national or state parties on the basis of their poll performance. The other parties are simply declared as
registered unrecognized parties.
• The recognition granted by the Commission to the parties determines their right to certain privileges like
allocation of the party symbols, provision of time for political broadcasts on the state-owned television and
radio stations, and access to electoral rolls.
• Every national party is allotted a symbol exclusively reserved for its use throughout the country. Similarly,
every state party is allotted a symbol exclusively reserved for its use in the state or states in which it is so
recognized. A registered-unrecognized party, on the other hand, can select a symbol from a list of free
symbols.
• Statement 1 is not correct: Election Commission of India (EC) has sought powers to deregister a political
party, and to be able to issue orders regulating registration and derecognition of political parties. The EC
contends that at present it only has very limited power to deregister a political party. EC can deregister a
political party under three circumstances:
o where a political party obtained registration by fraud
o if it is declared illegal by the Central government
o if a party amends its internal constitution and notifies the EC that it can no longer abide by the Indian
Constitution.
• Statement 2 is not correct: There is no bar on receiving donations by registered unrecognised political
parties. Recently, a report by Association for Democratic Reforms (NGO) presented several findings on
registered unrecognised political parties (RUPP). Their numbers have increased two folds from 1112 to
2301 in last 10 years. Individuals contributed 81% of donations to RUPP while corporates contributed 16%
between 2017-19.
• Statement 3 is correct: As per law there is no bar on any candidate to be proposer of another candidate for
the same constituency.

Q 59.B
• Zinc-finger nucleases (ZFNs) have had a central role in the development and application of genome
engineering technologies. These are based on zinc finger proteins, a family of naturally occurring
transcription factors, fused on an endonuclease. Zinc finger domains can recognize a trinucleotide DNA
sequence. A series of linked zinc finger domains can, therefore, recognize longer DNA sequences, providing
desired on-target specificity. It has wider application in stem cell therapy research, human genome project
(HGP) and has the potential to treat complex tumour surgeries. Hence statement (b) is correct answer.
20 www.visionias.in ©Vision IAS
Q 60.D
• Chromosomes: The nucleus has a double layered covering called nuclear membrane. The nuclear
membrane has pores which allow the transfer of material from inside the nucleus to its outside ,that is, to
the cytoplasm.
• The nucleus contains chromosomes which are visible as rod-shaped structures only when the cell is about
to divide. Chromosomes contain information for inheritance of features from parents to next generation in
the form of DNA molecules. Chromosomes are composed of DNA and protein. DNA molecules contain
the information necessary for constructing and organizing cells. Functional segments of DNA are called
genes. Hence, option (d) is the correct answer.
• In a cell which is not dividing, this DNA is present as part of chromatin material. Chromatin material is
visible as entangled mass of thread like structures. Whenever cell is to divide ,the chromatin material gets
organized into chromosomes.

Q 61.C
• Recent Context: A 22-year-old transwoman received a landmark verdict from Kerala High Court when the
court gave her relief to join the National Cadet Corps. The High Court not only allowed her to apply under
the female category but also asked the centre to allow entry of transgender persons in NCC within six
months. Hence the centre is bound to make changes to the prescribed enrollment criteria under Section 6 of
the NCC act, 1948. Hence, statement 2 is not correct.
• The National Cadet Corps (NCC) is a youth development movement. It has enormous potential for nation-
building. The NCC provides opportunities to the youth of the country for their all-round development with
a sense of Duty, Commitment, Dedication, Discipline and Moral Values so that they become able leaders
and useful citizens. NCC Programme is a part of the Educational activity, not of military activity. Hence,
statement 1 is not correct.
• The NCC is open to all regular students of schools and colleges on a voluntary basis. The students have no
liability for active military service. Age limit for a student to join NCC is:
o Junior Division/Wing (Boys/Girls) – 12 years to 18½ years
o Senior Division/Wing (Boys/Girls) – Upto 26 year
• The National Cadets Corps came into existence under the National Cadet Corps Act XXXI of 1948 (passed
in April 1948; came into existence on 16th July 1948).
• The Ministry of Defence deals with NCC at the National level. The Ministry of Education deals with NCC
in all States. Hence, statement 3 is correct.
• The funds are shared by the Central and State Govts.

Q 62.C
• Recent Context: Karnataka government will commence field research at Keshavapura in Karnataka to
solve the mystery regarding the birthplace of Purandara Dasa.
• Till now, it was believed that Purandara Dasa was born in Purandaragarh near Pune, Maharashtra. However,
an expert committee constituted by Karnataka Government reported that there is enough evidence to suggest
Keshavapura as his birthplace and recommended further research.
• About Purandara Dasa
o He (1484-1564) was a saint, poet and singer during the Vijayanagara empire.
o He was the disciple of Vyasatirtha, Rajaguru of the Vijayanagara empire.
o Prior to his initiation to the Haridasa tradition (music and literary tradition followed by
Vaishnavaits), Purandara Dasa was a rich merchant and was called Srinivasa Nayaka.
o He is regarded as the ‘Pitamaha’ of Carnatic music. His systemized method of teaching Carnatic
music is followed till the present day.
o He introduced the raga “Maya malava gowla” as the basic scale for music instruction.
o Under the pen name‘Purandara Vittala’, he has written around 4.75 lakh devotional songs in simple
Kannada, known as kirtans touching the local issues and traditions of South India.
o Swami Haridas, teacher of the foremost Hindustani musician Tansen, was a disciple of Purandara Dasa.
Hence option (c) is the correct answer.
Q 63.B
• Bhitauli means ‘to meet’. This auspicious festival is devoted to the married females who wait for the month
of Chaitra (the first month of the Hindu calendar) to celebrate the fest. It is celebrated in the Kumaon region
of Uttarakhand. The festival is celebrated as the parents of the married woman come to meet her with sweets
and gifts.
• Ramman is a religious festival manifested in the form of a ritual theatre held at the twin villages of Saloor-
Dungra in the Chamoli district. It is a religious festival of Uttarakhand that has received the honour of being
21 www.visionias.in ©Vision IAS
recognized by UNESCO in its Representative List of the Intangible Cultural Heritage of Humanity.
This Unique festival is not celebrated anywhere else in Uttarakhand. The festival and the eponymous
art form are conducted as an offering to the village deity, Bhumiyal Devta. It is celebrated usually on
the auspicious day of Sankranti in the month of Baisakh on the confluence of rivers Alaknanda and
Mandakini.
• Dasakathia is a traditional Odia folk art. It is a performing art widely performed by local or travelling
performers within the state of Odisha. Dasakathia performances mainly tell mythological stories, primarily
about Lord Shiva, but also other gods as well. One important aspect of the performance is satire and social
message. The performers play the Kathia, a percussion instrument made up of a pair of wooden pieces while
singing and vocalizing.
• Bhand Pather is an age-old traditional theatre in Kashmir. It is handed down from one generation to the
other in a guru-shishya parampara and thus does not have any distinct script. The storylines are often
humorous and satirical, and farce is an essential component of the plays.
• Hence option (b) is the correct answer.

Q 64.A
• Recent Context – Supreme Court will urgently hear a plea by NGO Association for Democratic Reforms
to stay the sale of a new set of electoral bonds on April 1, 2021.
• Electoral bond scheme was announced in Union Budget 2017-18 in an attempt to “cleanse the system of
political funding in the country.” The electoral bonds were introduced by amendments made through the
Finance Act 2017 to the Reserve Bank of India Act 1934, Representation of Peoples Act 1951, Income Tax
Act 1961 and Companies Act.
• Electoral Bond is an interest-free financial instrument for making an anonymous donation to political
parties. It resembles a promissory note and does not carry the name of the buyer or payee. Hence
statement 1 is correct.
• As per provisions of the Scheme, Electoral Bonds may be purchased by a person, who is a citizen of India
or incorporated or established in India. A person being an individual can buy bonds, either singly or jointly
with other individuals. A donor with a KYC-compliant account can purchase the bonds and can then donate
them to the party or individual of choice.
• The bonds shall be issued in the denomination of Rs. 1000, Rs. 10,000, Rs. 1,00,000, Rs.10,00,000 and Rs.
1,00,00,000. The State Bank of India (SBI) has been authorised to issue and encash Electoral Bonds through
its select Designated Branches.
• It will be available for purchase by any person for a period of ten days each in the months of January, April,
July and October as may be specified by the Central Government. An additional period of thirty days shall
be specified by the Central Government in the year of general elections to the House of People.
• Only the Political Parties registered under Section 29A of the Representation of the People Act, 1951
(43 of 1951) and which secured not less than one per cent of the votes polled in the last General
Election to the House of the People or the Legislative Assembly of the State, shall be eligible to receive
the Electoral Bonds. Hence statement 2 is not correct.
• The Election bonds can be encashed only by an eligible political party by depositing the same in their
designated bank account. The Electoral Bond deposited by an eligible Political Party in its account shall be
credited on the same day
• The Electoral Bonds shall be valid for fifteen calendar days from the date of issue and no payment shall be
made to any payee Political Party if the Electoral Bond is deposited after the expiry of the validity period.
The amount of bonds not encashed within the validity period of fifteen days shall be deposited by the
authorised bank to the Prime Minister Relief Fund.
• Electoral bonds cannot be traded on stock exchanges. They cannot be used as collateral for loans.
They are available only in physical form. Hence statement 3 is not correct.

Q 65.B
• The ease with which a government can reduce its debt-to-GDP ratio depends on its IRGD. IRGD is the
differential between interest rate and growth rate. If the IRGD is more negative then it is easier and
quicker for the government to ensure debt sustainability. Conversely, if the IRGD is positive, it is harder
and slower for the government to ensure debt sustainability. Hence, statement 1 is not correct.
• As a norm in India, over the last two and a half decade, GDP growth rate has been higher than the interest
rate. This has caused debt levels to decline. Hence, statement 2 is correct.
• Further, higher variability in the growth rates relative to interest rates over the same period implies that the
changes in IRGD are mostly due to changes in growth rates than interest rates. Thus, it is a higher growth
that provides debt sustainability in India.
22 www.visionias.in ©Vision IAS
Q 66.D
• Bitcoin is a decentralized, peer-to-peer, Cryptocurrency. Currently, RBI does not have the legal framework
to directly regulate cryptocurrencies. Hence, statement 4 is not correct.
• Bitcoin is mined using computational means. There are only 21 million bitcoins that can be mined in total.
This means there is an in-built system of deflation of Bitcoin currency as the total number of Bitcoins
are finite and limited. Hence, statement 1 is correct.
• Bitcoin wallet files that store the necessary private keys can be accidentally deleted, lost or stolen.
This is pretty similar to physical cash stored in a digital form. About 20 per cent of the 18.5 million
bitcoin in existence are reportedly stuck in lost or inaccessible digital wallets. Hence statement 2 is not
correct.
• Further, the system is designed to reward early adopters. Since each bitcoin will be valued higher with each
passing day, the question of when to spend becomes important. This might cause spending surges which
will cause the Bitcoin economy to fluctuate very rapidly, and unpredictably. Hence, statement 3 is not
correct.

Q 67.C
• Recently, China sold negative-yield debt for the first time, and this saw high demand from investors across
Europe.
• Negative-yield bonds are debt instruments that offer to pay the investor a maturity amount lower than the
purchase price of the bond. These are generally issued by central banks or governments, and investors pay
interest to the borrower to keep their money with them. Hence, statement 1 is correct and statement
2 is correct.
• Negative-yield bonds attract investments during times of stress and uncertainty as investors look to protect
their capital from significant erosion. Due to the Covid-19 pandemic and interest rates in developed
markets across Europe being much lower, investors are looking for relatively better-yielding debt
instruments to safeguard their interests.

Q 68.A
• Cairns Group is a coalition of 19 agricultural exporting countries (members) which account for more
than 25 per cent of the world’s agricultural exports, and one observer (Ukraine)
• It was established in Cairns, Australia, on 25-27 August 1986. It works for the liberalisation of global
trade in agricultural exports. It is Made up of developed and developing countries from six continents
• Members of the Group are Argentina, Australia, Brazil, Canada, Chile, Colombia, Costa Rica,
Guatemala, Indonesia, Malaysia, New Zealand, Pakistan, Paraguay, Peru, the Philippines, South Africa,
Thailand, Uruguay and Viet Nam. Ukraine is an observer of the Group.
• India is not a member of the group. Hence option (a) is the correct answer.

Q 69.A
• Recently it was reported that the government is mulling putting in place a ‘Treasury Single Account’ for all
Central ministries and departments to help lower the cost of borrowing and enhance efficiency in fund
flows. Pilot is on to implement Single Treasury Account for some autonomous bodies. Based on the
outcome, the effort would be to use the system for all the Central Government ministries and departments.
• Under the existing process, after approval, funds are allocated and disbursed to various ministries,
departments, autonomous bodies and States. It has been observed that funds are not being utilised and kept
idle in other bank accounts. Indian budget makes a provision for borrowing to meet part of the expenditure
as revenue is not sufficient. Given the borrowing cost, such a system puts further strain on the exchequer
and necessitates ways for innovation. One such innovation is Treasury Single Account or TSA.
• According to the International Monetary Fund (IMF) Working Paper, a TSA is a unified structure of
government bank accounts that gives a consolidated view of government cash resources. Based on the
principle of unity of cash and the unity of treasury, a TSA is a bank account or a set of linked accounts
through which the government transacts all its receipts and payments. The principle of unity follows from
the fungibility of all cash irrespective of its end use.
• The primary objective of a TSA is to ensure effective aggregate control over government cash balances.
The consolidation of cash resources through a TSA arrangement facilitates government cash management
by minimising borrowing costs. Hence option (a) is the correct answer.

23 www.visionias.in ©Vision IAS


Q 70.D
• Simbalbara National Park is a national park located in sirmour district Himachal Pradesh. It is located on
the foothills of the Himalayas in the Shivalik region. Hence it is not part of the greater Himalayas.
• Pin Valley National Park is located in the Lahaul & Spiti district of Himachal Pradesh. It lies at an extension
of greater Himalayan to trans-Himalayan. The region is a cold desert, which is adjoined to the Great
Himalayan National Park in the southwest and Rupi Bhabha sanctuary in the south. Hence it is the part of
the greater Himalayas
• Hemis National park lies within the Karakoram West Tibetan plateau alpine steppe ecoregion which is
basically a trans-Himalayan region located in Laddakh. It is also the only national park of India that is is in
the north of the Himalayas. Therefore it does not lie in the greater Himalayan region.
• Singalila National Park is located on Singalila Ridge in the Eastern Himalayan region in Darjeeling district.
It is the part of the lesser Himalayas. Sandakphu and Phalut, the two of the highest peaks in West Bengal,
are located right inside the park. Therefore it does not lie in the greater Himalayan region.
• Hence options 1, 3, and 4 are correct.

Q 71.A
• GRACE (Gravity Recovery and Climate Experiment), is a joint US-German satellite(NASA and German
Aerospace Centre) mission that has provided new and unexpected insights into the natural processes of
the Earth. GRACE Follow-On is a successor of the GRACE observes the critical indicators of climate
change through changes in Earth's gravitational field using LISA technology. Hence statement 2 is
not correct.
• GRACE Follow-On (GRACE-FO) continues the legacy of GRACE, tracking Earth’s water movement
and surface mass changes across the planet. Monitoring changes in ice sheets and glaciers, near-surface
and underground water storage, the amount of water in large lakes and rivers, as well as changes in sea level
and ocean currents provides an integrated global view of how Earth’s water cycle and energy balance
are evolving—measurements that have important applications for everyday life. Hence statement 1
is correct.

Q 72.A
• The Indian Council of Agricultural Research has announced the KRITAGYA (Krishi-Raknik-Gyan)
hackathon under the National Agricultural Higher Education Project (NAHEP). Hence, statement 2 is
not correct.
• The hackathon has been introduced with an aim to promote the potential technology solutions for
enhancing the farm mechanization with special emphasis on women-friendly equipment. Hence,
statement 1 is correct.
• The students participating in the KRITAGYA hackathon can collaborate with the local start-ups, students
from technology institutes can win cash prize.
• KRITAGYA hackathon will provide the students, faculties, entrepreneurs, innovators, and other
stakeholders an opportunity to showcase their innovative approach and technology solutions to promote
farm mechanization in India. The programme will also help in enhancing the learning capabilities,
innovation and disruptive solutions, employability, and entrepreneurial drive in the Farm Mechanization
Sector. He further added that the event will also help in taking forward the vision of high-quality higher-
education with equity and inclusion as envisaged in the NEP 2020.

Q 73.C
• Recent Context: Recently, the Reserve Bank of India (RBI) released National Strategy for Financial
Education (NSFE): 2020-2025 document in order to promote financial inclusion in the country. Hence
statement 1 is not correct.
• Strategic Objectives of the NSFE are:
o to promote financial literacy concepts among the various sections of the population through
financial education to make it an important life skill
o Improve usage of digital financial services in a safe and secure manner
o encouraging active savings behaviour
o Encourage participation in financial markets to meet financial goals and objectives
o Improve usage of digital financial services in a safe and secure manner
• In order to achieve the Strategic Objectives laid down, the document recommends adoption of a ‘5 C’
approach for dissemination of financial education through emphasis on
o development of relevant Content (including Curriculum in schools, colleges and training
establishments), developing Capacity among the intermediaries involved in providing financial
24 www.visionias.in ©Vision IAS
services, leveraging on the positive effect of Community led model for financial literacy through
appropriate Communication Strategy, and lastly, enhancing Collaboration among various
stakeholders. The implementation process will be directly monitored by the Technical Group on
Financial Inclusion and Financial Literacy (TGFIFL) (Chair: Deputy Governor, RBI). Hence
statement 3 is correct.
o It is the second National Strategy for Financial Education (NSFE) for the five years (2020-25) as
the first National Strategy for Financial Education (NSFE) in 2013 was adopted for the period 2013-
2018 after consulting different stakeholders ( (viz. RBI, SEBI, IRDAI and PFRDA). Hence statement
2 is not correct.

Q 74.D
• Supplementary Grant- It is granted when the amount authorized by the Parliament through the
appropriation act for a particular service for the current financial year is found to be insufficient for that
year. Hence pair 1 is correctly matched.
• Additional Grant – granted when a need has arisen during the current financial year for additional
expenditure upon some new service not contemplated in the budget.
• Excess Grant – granted when money has been spent on any service during a financial year in excess of the
amount granted for that service in the budget for that year. Hence pair 2 is correctly matched.
• Token Grant- It is granted when funds to meet the proposed expenditure on a new service can be made
available by reappropriation. Demand for the grant of a token sum (of Re.1/-) is submitted to vote of the
Lok Sabha and if assented, funds are made available. Hence pair 3 is not correctly matched.
• Exceptional Grant – granted for a special purpose and forms no part of the current service of any financial
year.
• Vote of Credit – granted for meeting an unexpected demand upon the resources of India, when on account
of the magnitude or the indefinite character of the service, the demand cannot be stated with the details
ordinarily given in a budget. Hence pair 4 is correctly matched.
Q 75.C
• Immediately after the acquisition of political power in India, the company officials wanted to maintain
neutrality in the sphere of religion and culture of the indigenous society. However, constant pressure from
different quarters, the Missionaries, the Liberals, the Orientalists, the Utilitarians compelled the company
to give up its policy of neutrality and to take the responsibility of promotion of learning. Opinions were
sharply divided was whether the company should promote western or oriental learning. In the initial stage,
the company officials patronized oriental learning.
• Asiatic Society of Bengal was founded in January 1784 by William Jones and thirty other members who
had responded to his call for pursuing various branches of Asiatic studies. Membership was voluntary but,
until 1829, no Indians were admitted. The first volume of its publication, Asiatic Researches, was brought
out in 1789.
• Calcutta Madrasa was set up by Warren Hastings in 1781 for the study and teaching of Muslim law and
related subjects, and the Sanskrit College at Varanasi by Jonathan Duncan in 1792 for the study of
Hindu law and philosophy (both were designed to provide a regular supply of qualified Indians to help the
administration of law in the courts of the Company).
• Those who were in favour of the continuation of the existing institutions of oriental learning and promotion
of Indian Classical tradition were called Orientalists. Orientalists also wanted to teach the British officials
the local language and culture so that they would be better at their job. This was the prime objective behind
the foundation of the Fort William College at Calcutta in 1800.
• However, the Macaulay’s Minute on Education in 1835 marked the victory of the so-called ‘Anglicists’
as well as ‘progressive’ Indians, who supported the introduction and popularisation of English education
over the opposing school of thought represented by the ‘Orientalists’ who preferred to encourage the pursuit
of traditional learning.
• Hence the correct order is 2-4-1-3.

Q 76.D
• The rhizosphere is also known as the microbe storehouse is the soil zone surrounding the plant roots where
the biological and chemical features of the soil are influenced by the roots. The rhizosphere is coined more
than a hundred years ago by Hiltner in 1904. It is a hot spot for microorganisms, where severe, intense
interactions take place between the plant, soil, and microfauna.
• It is a nutrient-rich region of the soil immediately surrounding the plant root. It is a dynamic region governed
by complex interactions between plants and the organisms that are in close association with the root.
• Hence, statement 1 is not correct.
25 www.visionias.in ©Vision IAS
• The rhizosphere is the most active portion of the soil in which biogeochemical processes influence the root
system of plants and in turn, get influenced by the roots of plants.
• The rhizosphere can be divided into three zones:
o Endorhizosphere
o Rhizoplane
o Ectorhizosphere
• Beneficial or harmful relationships exist between rhizosphere organisms and plants, which ultimately affect
root function and plant growth.
• The rhizosphere harbours a diverse reservoir of culturable microorganisms that can be exploited to benefit
mankind. Many rhizosphere microbes benefit crop production, reducing the dependence on chemical
fertilizers to achieve high productivity.
• While many members of the rhizosphere microbiome are beneficial to plant growth, also plant pathogenic
microorganisms colonize the rhizosphere striving to break through the protective microbial shield and to
overcome the innate plant defense mechanisms in order to cause disease.
• While many members of the rhizosphere microbiome are beneficial to plant growth, there are also
plant pathogenic microorganisms that colonize the rhizosphere and can cause diseases in plants.
Hence, statement 2 is not correct.
• There are also potential human pathogenic microorganisms in this nutrient-enriched plant-soil environment
which can cause potential public health hazards for the people.

Q 77.D
• An impartial and independent judiciary protects the rights of the individual and provides equal justice
without fear and favor. The constitution of India provides many privileges to maintain the independence of
the judiciary.
• Judicial accountability is a corollary fact of the independence of the judiciary. Simply put, accountability
means taking responsibility for your actions and decisions. Generally, it means being responsible to any
external body; some insist accountability to principles or to oneself rather than to any authority with the
power of correction or punishment.
• Independence of the judiciary does not imply arbitrariness or absence of accountability. Judiciary is a part
of the democratic political structure of the country. It is therefore accountable to the Constitution, to
the democratic traditions, and to the people of the country.
• Hence option (d) is correct answer.

Q 78.D
• Catastrophe Containment and Relief Trust was designed in 2015 during the Ebola outbreak to provide
relief during catastrophic natural or public health disasters and free up resources to meet the exceptional
balance of payments needs. In 2020, its eligibility criteria were relaxed in response to Covid-19 and the
instrument was initially approved for 25 eligible countries. Hence statement 1 is not correct.
• The Catastrophe Containment and Relief Trust (CCRT) allows the IMF to provide grants for debt relief
for the poorest and most vulnerable countries hit by catastrophic natural disasters or public health
disasters. The relief on debt service payments frees up additional resources to meet the exceptional balance
of payments needs created by the disaster and for containment and recovery. Hence statement 2 is not
correct

Q 79.A
• Policy Cut: That the amount of the demand be reduced to Re.1/-'. It represents the disapproval of the
policy underlying the demand. A member giving notice of such a motion shall indicate in precise terms the
particulars of the policy which he proposes to discuss. The discussion shall be confined to the specific point
or points mentioned in the notice and it shall be open to members to advocate an alternative policy. If a
policy cut motion is adopted by Parliament and the government does not have the numbers, it is obliged to
resign as per rules of the Lok Sabha. Hence Option a is the correct answer.
• Calling Attention Motion: It is an innovation in the modern parliamentary procedure and combines the
asking of a question for answer with supplementaries and short comments in which different points of view
are expressed concisely and precisely, and the Government has adequate opportunity to state its case. It
gives members an opportunity to bring to the surface the failure or inadequate action of Government on a
matter of urgent public importance. This procedural device is analogous to an adjournment motion without
its censure aspect.
• Economy Cut: That the amount of the demand be reduced by a specified amount' (which may be either
a lumpsum reduction in the demand or omission or reduction of an item in the demand). The notice shall
26 www.visionias.in ©Vision IAS
indicate briefly and precisely the particular matter on which discussion is sought to be raised and speeches
shall be confined to the discussion as to how economy can be effected
• Token Cut: That the amount of the demand be reduced by Rs.100/- in order to ventilate a specific
grievance which is within the sphere of the responsibility of the Government of India. The discussion
thereon shall be confined to the particular grievance specified in the motion.

Q 80.A
• Galathea National Park is a National Park located in the Union Territory of Andaman and Nicobar Islands,
India. It is located on the island of Great Nicobar in the Nicobar Islands. The total area of this park is
some 110 square kilometers, and it was gazetted as a National Park of India in 1992. Galathea forms part
of what has been designated as the Great Nicobar Biosphere Reserve, which also includes the larger
Campbell Bay National Park, separated from Galathea by a 12-km forest buffer zone. The Galathea National
Park comprises the core area of the southern part of the Biosphere Reserve in Great Nicobar Island and
consists of lowland forests that have the greatest abundance of endemic avifauna.
• Rani Jhansi Marine National Park lies within Ritchie’s Archipelago, 14 km northeast of South
Andaman Islands and southeast of Middle Andaman Island. The natural habitats of the park include
lagoons, coral reefs, beaches, lowland evergreen rain forest, semi-evergreen rain forest, and mangrove
forests.
• Mount Harriet National Park is a national park located in the Andaman and Nicobar Islands union
territory of India. The park, established in 1969, covers about 4.62 square km. Mount Harriet, which is part
of the park, is the third-highest peak in the Andaman and Nicobar archipelago next to Saddle Peak.
• Saddle Peak is located on the Northern side of the Andaman Islands, The species Scolopia Pusilla and
Cliestanthus robustus (a wild plant) are found here that are not found in mainland India. Hence option (a)
is the correct answer.
Q 81.D
• Recent Context: Researchers at the Indian National Centre for Ocean Information Services (INCOIS),
Hyderabad have reportedly found a way to better forecast waves based on Boreal Summer Intra Seasonal
Oscillation (BSISO).
• Statement 1 is correct: BSISO is the movement of convection (heat) from the Indian Ocean to western
Pacific Ocean roughly every 10-50 days during the monsoon (June-September).
• Statement 2 is correct: They represent the monsoon’s ‘active’ and ‘break’ periods, in which weeks of
heavy rainfall give way to brilliant sunshine before starting all over again. The active phase also enhances
monsoon winds and hence the surface waves.
• Statement 3 is correct: Some phases of boreal summer intraseasonal oscillation or BSISO induce high
wave activity in the north Indian Ocean and Arabian Sea, the researchers claimed. The will help improve
wave forecasts along India’s coasts and mitigate adverse impacts of high waves (coastal flooding, erosion,
etc). Sea-navigation routes can also be better planned.
• Waves induced by active phases of BSISO are nearly 0.5 meters higher than those which occur during other
phases of BSISO. The active phases of BSISO occur between June and August which are the monsoon
summer months.
• It has a great significance in developing seasonal and climate forecast services for waves and coastal erosion
for India.

Q 82.D
• Criminalization of Politics implies that the criminals are entering the election fray and contesting elections
and even getting elected to the Parliament and state legislature.
• Some of the steps taken by the Election Commission (EC) to De-Criminalize Indian Politics are:
o EC directed all the Returning Officers (ROs) to reject the nomination papers of any candidate who
stands convicted on the day of filing the nomination papers even if his sentence is suspended.
o A system of flying squads has been introduced to seize black money during elections.
o It carried out a much more intense voter awareness campaign and even initiated a campaign using
celebrities exhorting voters not to sell their vote.
o A candidate to any National or State Assembly elections is required to furnish an affidavit containing
information regarding their criminal antecedents, if any, their assets, liabilities, and educational
qualification.
• Limitations of EC in tackling Criminalization
o No power to disqualify candidates prior to conviction even if a person is facing several serious
charges. Section 8 of the RPA, 1951 deals with disqualification only after a person is convicted for
certain offences.
27 www.visionias.in ©Vision IAS
o Matters of election false affidavits can have extremely serious consequences as it affects the purity of
elections. Lax punishments have not been able to deter this activity. Also false affidavit or suspension
of material information in the affidavit is not included as grounds for challenging the election or
for rejection of nomination papers section under RPA, 1951.
o Misuse of religion for electoral gain: While such practices are qualified as corrupt practices, they can
be questioned only by way of an election petition and cannot be a subject of enquiry before the EC
when the election is in progress.
o Ironically these provisions will have application only during the period of election and there is no
provision to challenge the corrupt practice of the candidate who lost the election.
o Law enforcing authorities cannot proceed without a warrant issued by a competent Magistrate to search
or arrest any person even on specific information about the corruptive practice. These provide the
violators an opportunity to evade legal action.
• Hence, under the above circumstances, a contesting candidate can not be disqualified during the
election.

Q 83.D
• Sulphur dioxide is a colourless, soluble gas with a characteristic pungent smell that forms sulphuric acid
when combined with water.
• India is the largest emitter of sulphur dioxide (SO2) in the world, contributing more than 15 percent of
global anthropogenic emissions, according to a new report by Greenpeace released on August 19, 2019.
• The primary reason for India’s high emission output is the expansion of coal-based electricity
generation over the past decade, the report added. Five of the top 10 SO2 emission hotspots from the
coal/power generation industry across the world are in India,
• Sulphur dioxide is produced mainly from the combustion of fossil fuels that contain sulphur, such as coal
and oil (eg, coal being burnt in a home fireplace for heating and diesel-powered vehicles). Sulphur dioxide
is also produced from some industrial processes (eg fertilizer manufacturing, aluminum smelting, and
steel-making).
• Natural sources of sulphur dioxide include geothermal activity. Sulphur dioxide can be formed naturally
by volcanoes, forest fires, etc.
• Hence option (d) is the correct answer.

Q 84.A
• Proton Therapy is a highly sophisticated form of Radiation Therapy that uses high-energy protons to target
tumours accurately while greatly reducing the radiation dosage to the surrounding healthy tissues. The
precision of proton therapy allows it to preserve vital structures at the tumor site, thereby giving myriad
benefits to the patient. One of the most significant advantages of Proton Therapy is the reduced side effects
and enhanced quality of life for a patient during and after treatment. Hence statement 1 is correct.
• It uses protons rather than x-rays to treat cancer. At high energy, protons can destroy cancer cells. It can
also be combined with x-ray radiation therapy, surgery, chemotherapy, and/or immunotherapy. Like x-ray
radiation, proton therapy is also a type of external-beam radiation therapy. Hence statement 2 is not
correct.

Q 85.A
• A Power Purchase Agreement (PPA) often refers to a long-term electricity supply agreement between
two parties, usually between a power producer and a customer (an electricity consumer or trader).
Hence, statement 1 is correct.
• The PPA defines the conditions of the agreement, such as the amount of electricity to be supplied, negotiated
prices, accounting, and penalties for non-compliance. Since it is a bilateral agreement, a PPA can take many
forms and is usually tailored to a specific application. PPAs are typically of 25-30 years duration.
• Currently, PPAs with central generating stations neither have an option to exit, nor a review provision.
Ministry of Power in December 2020, proposed to introduce flexibility in power purchase after the
completion of 25 years PPAs.
• Electricity Contract Enforcement Authority, a Central Enforcement Authority headed by a retired
Judge of the High Court is proposed under Electricity Act (Amendment) Bill 2020 to be set-up with
powers of the Civil Court to enforce performance of contracts related to purchase or sale or
transmission of power between a generating, distribution or transmission companies.
• Central Electricity Regulatory Commission does not deal with the enforcement of PPAs. Hence, statement
2 is not correct.

28 www.visionias.in ©Vision IAS


Q 86.C
• The Model Code of Conduct is a set of norms and guidelines which is laid down by the Election
Commission for political parties and candidates which has been evolved with the consensus of
political parties.
• This is in keeping with Article 324 of the Constitution, which gives the Election Commission the power for
conducting free, fair, and peaceful elections to the Parliament and the State Legislatures. Hence statement
1 is correct
• The Model Code of Conduct is enforced from the date of announcement of the election schedule by the
Election Commission and is operational till the process of elections is completed. Hence statement 2
is correct.
• Applicability of MCC during general elections and bye-elections?
o During general elections to the House of People (Lok Sabha), the code is applicable throughout the
country.
o During general elections to the Legislative Assembly (Vidhan Sabha), the code is applicable in the
entire State.
o During bye-elections, in case the constituency is comprised in State Capital/Metropolitan
Cities/Municipal Corporations, then the code would be applicable in the area of concerning
Constituency only. In all other cases, the MCC would be enforced in the entire district(s) covering the
Constituency going for bye-election(s). Hence statement 3 is correct.
• However, there is no statutory backing for the implementation of MCC and It is not enforceable by
law. Though, certain provisions of the MCC may be enforced through invoking corresponding
provisions in other statutes such as the Indian Penal Code, 1860, Code of Criminal Procedure, 1973,
and Representation of the People Act, 1951. Earlier, The Election Commission has argued against
making the MCC legally binding but currently, it is legally non-binding in nature. Hence statement 4 is
not correct.

Q 87.B
• Hydroponics is a production method where the plants are grown in a nutrient solution rather than in soil.
Although almost any crop can be grown hydroponically, the most common are leaf lettuce, tomatoes,
peppers, cucumbers, strawberries, watercress, celery and some herbs. Hence statement 1 is correct.
• Plants grown hydroponically, grow faster and healthier than plants grown in the soil as necessary nutrients
are provided directly to the roots through the water.
• Nutrients are dissolved in water that surrounds the roots, so plants have easier access to the nutrition
they need and spraying or applying essential nutrients directly to plant leaves not required. Hence
statement 2 is not correct.
• It was even integrated into the space program by NASA, as they are considering the practicality of growing
foods in a long term space mission.
• Advantages:
o Fresh organic produce can be harvested throughout the year.
o Higher yields of produce than traditional soil-based agriculture and it allows crops to grow in areas that
can not support crops in soil.
o Greater plant density and it can be done indoor.
o Plants can be moved as they grow.
o Less water consumption- In methods where the root system is contained in a closed trough or tube, less
evaporation occurs and water consumption is reduced.
o It requires only 10% of the water that soil-based agriculture requires.
o The water used in the hydroponic system can be reused, thereby reducing the need for a more freshwater
supply.
o It is good for the environment and grown products since it eliminates the need for massive pesticide use
compared to traditional soil-based agriculture.
o Hanging Gardens of Babylon and floating garden of Aztecs of Mexico are an example of Hydroponic
culture.

Q 88.A
• State Development Loans (SDLs) are dated securities issued by states for meeting their market borrowings
requirements. In effect, the SDL are similar to the dated securities issued by the central government. Purpose
of issuing State Development Loans is to meet the budgetary needs of state governments. They are
accounted for in the budget of the state. Each state can borrow upto a set limit through State Development
Loans. Hence statement 1 is correct.
29 www.visionias.in ©Vision IAS
Q 93.A
• Trade receivable is the balance of money due to a firm for goods or services delivered or used but not yet
paid for by customers. Trade receivable discounted refers to the selling of unpaid outstanding invoices for
a cash amount that is less than the face value of those invoices. Trade receivables are listed on the balance
sheet as a current asset.
• Trade Receivables Discounting System (TReDS) is an electronic platform for facilitating the
financing / discounting of trade receivables of Micro, Small and Medium Enterprises (MSMEs)
through multiple financiers. These receivables can be due from corporates and other buyers, including
Government Departments and Public Sector Undertakings (PSUs). Hence, statement 1 is correct.
• Sellers, buyers and financiers are the participants on a TReDS platform. Only MSMEs can participate as
sellers in TReDS. Corporates, Government Departments, PSUs and any other entity can participate as
buyers in TReDS. Banks, NBFC - Factors and other financial institutions as permitted by the Reserve Bank
of India (RBI), can participate as financiers in TReDS. Hence, statement 2 is correct.
• One of the major reasons for the introduction of TReDS is to ensure timely payments to micro, small and
medium enterprises (MSMEs), allowing them to access credit at an interest rate which would be offered to
their buyers. This provides MSMEs an alternate source of creating credit history.
• One of the biggest advantages of TReDS platform for MSMEs is the fact that the funds disbursed through
TReDS are without recourse or collateral. In the case of non-payment by the corporate buyer, the financier
would not claim damages from the MSME.
32 www.visionias.in ©Vision IAS
• Receivables Exchange of India Ltd (RXIL) is a joint venture between Small Industries Development Bank
of India (SIDBI) – the apex financial institution for promotion and financing of MSMEs in India and
National Stock Exchange of India Limited (NSE). RXIL operates the Trade Receivables Discounting
System (TReDS) Platform as per the TReDS guideline issued by Reserve Bank of India (RBI). Hence,
statement 3 is not correct.

Q 94.C
• The National Legal Services Authority (NALSA) has been constituted under the Legal Services Authorities
Act, 1987 (Hence statement 1 is correct) to provide free Legal Services to the weaker sections of the
society. Hence statement 2 is not correct.
• The sections of the society as enlisted under Section 12 of the Legal Services Authorities Act are entitled
for free legal services, they are :
o A member of a Scheduled Caste or Scheduled Tribe;
o A victim of trafficking in human beings or begar as referred to in Article 23 of the Constitution;
o A woman or a child;
o A mentally ill or otherwise disabled person;
o A person under circumstances of undeserved want such as being a victim of a mass disaster, ethnic
violence, caste atrocity, flood, drought, earthquake or industrial disaster; or
o An industrial workman; or
o In custody, including custody in a protective home within the meaning of clause (g) of Section 2 of the
Immoral Traffic (Prevention) Act, 1956(104 of 1956); or in a juvenile home within the meaning of
clause(j) of Section 2 of the Juvenile Justice Act, 1986 (53 of 1986); or in a psychiatric hospital or
psychiatric nursing home within the meaning of clause (g) of Section 2 of the Mental Health Act,
1987(14 of 1987);or
o a person in receipt of annual income less than the amount mentioned in the following schedule (or any
other higher amount as may be prescribed by the State Government), if the case is before a Court other
than the Supreme Court, and less than Rs 5 Lakh, if the case is before the Supreme Court.
• The Chief Justice of India is the Patron-in-Chief and the Senior most Hon'ble Judge, Supreme Court of India
is the Executive Chairman of the authority. Hence statement 3 is not correct.
• Free legal aid is the provision of free legal services in civil and criminal matters for those poor and
marginalized people who cannot afford the services of a lawyer for the conduct of a case or a legal
proceeding in any Court, Tribunal, or Authority. Hence statement 4 is correct.

Q 95.C
• The State of the World’s Forests 2020 which is given by UNEP and FAO assesses progress to date in
meeting global targets and goals related to forest biodiversity and examines the effectiveness of policies,
actions, and approaches, in terms of both conservation and sustainable development outcomes. Forests
cover 31 percent of the global land area. Since 1990, it is estimated that 420 million hectares of forest have
been lost through conversion to other land uses, although the rate of deforestation has decreased over the
past three decades. It was 16 million hectares per year in the 1990s while between 2015 and 2020 it was
estimated at 10 million hectares per year. Hence statement 2 is correct.
• The largest share of the protected area of the forest is found in South America and the lowest in
Europe. Hence statement 1 is not correct.
• The report mentions that Aichi Biodiversity Target 11 to protect at least 17% of terrestrial area by
2020, has been exceeded for forest ecosystems as a whole as 18% of the World's forest area falls within
legally established protected areas such as national parks, conservation areas (IUCN categories I-V).
Hence statement 3 is correct.
Q 96.C
• Inequality and income per capita have a converging relationship with socio-economic outcomes in India.
While considering the inequalities, in most cases, inequality of opportunity is much more objectionable than
inequality of outcomes, as individuals' opportunities are influenced by endowments that are related to
parents and other adults, peers, and a variety of chance occurrences throughout their lifetimes. Hence,
statement 1 is correct.
• Perfect equalisation of outcomes ex-post, i.e., after the efforts have been exerted to obtain those outcomes,
can reduce individuals’ incentives for work, innovation and wealth creation. Hence, statement 2 is correct.
• A benevolent social planner seeks to maximize aggregate welfare: an economy in which each individual
possesses 2 units of wealth is preferable to one in which each individual possesses only 1 unit of wealth.
This is true even if the planner assigns greater weight to the poor than the rich, i.e., the planner’s social
welfare function depends on not just the size of the pie but also how it is distributed.
33 www.visionias.in ©Vision IAS
• For a developing country such as India, where the growth potential is high and the scope for poverty
reduction is also significant, a policy that lifts the poor out of poverty by expanding the overall pie is
preferable as redistribution is only feasible if the size of the economic pie grows rapidly.

Q 97.B
• Representation of Peoples Act 1950 (RPA Act 1950) provides for the following:
o Qualification of voters. Hence option 1 is correct
o Preparation of electoral rolls. Hence option 4 is correct.
o Delimitation of constituencies.
o Allocation of seats in the Parliament and state legislatures.
o The manner of filling seats in the Council of States to be filled by representatives of Union territories.
o Officers (ex-Chief Electoral Officer etc.)
• Representation of People's Act, 1951 contains provisions relating to the following electoral matters:
o Qualifications and disqualifications for membership of Parliament and State Legislatures
o Notification of General Elections
o Administrative machinery for the conduct of elections
o Registration of political parties
o Conduct of elections
o Free supply of certain material to candidates of recognized political parties
o Disputes regarding elections
o Corrupt practices and electoral offenses

Q 98.A
• National Commission for Minority Educational Institutions (NCMEI), a quasi-judicial body, regulates the
certification of minority educational institutions all over India. Hence, statement 1 is correct.
• Its Chairman should be who has been a Judge of the High Court and three members are to be nominated by
Central Government. Hence, statement 3 is not correct.
• It has the powers of a Civil Court. It has both original and appellate jurisdiction in such matters, as laid
down by the SC in Joseph of Cluny v/s The State of West Bengal case.
• Commission has adjudicatory functions and recommendatory powers.
• It decides on disputes regarding the affiliation of a minority educational institution to a university.
• It has the power to enquire, suo motu, into complaints regarding deprivation or violation of rights of
minorities to establish and administer educational institutions of their choice.
• It specifies measures to promote and preserve the minority status and character of institutions of their choice
established by minorities.
• It can also cancel the minority status granted to institutions if they are found to have violated the conditions
of the grant. Issues of Linguistic minority institutions (LMIs).
• National Commissioner for Linguistic Minorities (NCLM)
o It was established under Article 350B of the Constitution.
o It reviews safeguards for the protection of linguistic minority rights and make recommendations to the
parliament based on its findings. Hence, statement 2 is not correct.

Q 99.B
• Sirocco is a warm, dry, and dusty local wind that blows in a northerly direction from the Sahara Desert
and after crossing over the Mediterranean sea reaches Europe. Sirocco becomes very strong and active at
test cyclonic origin time of the origin of cyclonic storms over the Mediterranean sea. It becomes extremely
warm and dry while descending through the northern slopes of the Atlas mountain.
• There are different local names for sirocco in Africa. e.g. Khamsin in Egypt, gibli in Libya, chilli in
Tunisia, etc. The warm and dry dusty winds in the Arabian Desert are called ‘Simmon's. Sirocco, while
passing over the Mediterranean Sea picks up moisture and yields rainfall in the southern part of Italy where
the sands with falling rains associated with sirocco is called ‘blood rain’ because of the fallout of red sands
with falling rains. Hence, option (b) is the correct answer.
• It may be remembered that sirocco while blowing through the Sahara desert picks up red sands which
settle down with rains in south Italy. It is apparent that sirocco is very much injurious to agricultural and
fruit crops.

34 www.visionias.in ©Vision IAS


Q 100.B
• Inter-operable Criminal Justice System (ICJS) is a common platform for information exchange and
analytics of all the pillars of the criminal justice system comprising of Police, Forensics, Prosecution,
Courts& Prisons.
• ICJS enables a nationwide search on police, prisons & courts databases across all States/ UTs in the
country. Therefore, ICJS takes data sharing to a higher level and ensures a single source of truth
between law enforcement and judicial systems.
• Hence statement 1 is correct.
• It is a key modernization programmes of the Union Ministry of Home Affairs (MHA). Hence
statement 2 is not correct.
• With the aid of the ICJS platform, the metadata of FIR and charge sheet can be accessed by all the High
Courts and subordinate courts. Documents like FIR, case diary and charge sheet are uploaded by police
in PDF format for utilization by the courts. Hence statement 3 is correct.
• To ensure effective implementation of ICJS in each State, High Courts have been requested to engage
the services of an IPS officer, who will be instrumental for the integration of data on the ICJS
platform.
• High Courts are also requested to appoint one Nodal Officer to ensure that apart from the police, other State
functionaries like Provident Fund Organisation, Forest Department, Municipal Authorities, Labour Welfare
Boards Town Planning Authorities and Food and Drug Administration are also part of ICJS.

35 www.visionias.in ©Vision IAS

You might also like